1st series [1] [2] [3] [4] [5] [6] [7] [8] [9] [10] [11] [12] [13] [14] [15] [16] [17] [18] [19] [20] [21] [22] [23] [24]  2nd series [1] [2] [3] [4] [5] [6] [7] [8] [9] [10] [11] [12] [13] [14] [15] [16] [17] [18] [19] [20] [21] [22] [23] [24] [25] [26] [27] [28] [29] [30] [31] [32] [33] [34] [35] [36] [37] [38] [39] [40] [41] [42] [43] [44] [45] [46] [47] [48] [49]

  View the latest questions and answers at askaphilosopher.org

Ask a Philosopher: Questions and Answers 27 (2nd series)

This is the twenty-seventh answers page to appear in the new format.

When referring to an answer on this page, please quote the page number followed by the answer number. The first answer on this page is 27/1.

(1) Leslie asked:

I intend to study Philosophy with the Open University, I have already studied the Intro to the Humanities, previous career was with the British Infantry and I am 47 years old.

My question is why do, when I mention I would like to study this subject to friends, family and former colleagues they either guff or hit the roof, what's the problem here?

---

Many people feel threatened by things they do not understand. Those people usually hate and distrust any ambition that goes beyond their own.

Rhonald Blommestijn

back

(2) Nicola asked:

What is death?

---

This may seem obvious, but if I 'die' that means that I lose consciousness and 'never wake up'. If I did wake up — at any time, even billions of years, in the future — then I didn't really die, I merely lost consciousness for a long while.

But think of what that means. To assert, 'GK has died' implies a statement about all future times, to infinity. There is no finite length of time such that, if 'you' were to reappear after ceasing to 'live', that person could not, as a matter of logic, be you. The problem is that I don't know what 'infinity' means. I don't understand, I can't get my mind around, the notion of infinite future time. But understanding the notion of infinite future time is a necessary condition for grasping what it is to 'die'.

If, like me, you do not understand the notion of infinite future time then, like me, you have no alternative but to admit that you do not know 'what death is', and no amount of talk about mind and body or the criteria for personal identity can make good that deficiency.

Geoffrey Klempner

back

(3) Douna asked:

Hey, I've been into a million discussions with people about love and I would appreciate it if you can please tell me what is love when you can't prove it with your senses? how can I convince others that it is an illusion that humans create?

---

One can answer this from the perspective of biology: a mother loves her child because it is the only way her genes will survive. Those species that don't love their children face extinction. Romantic love can be emplaned in similar terms: human infants need years of care from two parents. Already researchers claim they can find areas in the brain that provide a religious experience when stimulated. It makes it likely that such an area or a chemical substance will be found that triggers love. But: will that make love more real or less real for you? That depends on whether you want to understand or experience the phenomena.

Rhonald Blommestijn

back

(4) Mohamed asked:

Hello!

I'm quite embarrassed by the question: what is philosophy good for? (theme of utility of philosophy nowadays).

I just want to get a convincing answer to this embarrassing question. Have you any reference to philosophical texts or any other resources that contain answers to this question?

---

It won't make your richer or healthier. It might make you happier when you understand things that before were mysteries to you. And most useful of all: it can make it a lot easier to spot nonsense and fake arguments. Study philosophy and you will find there is a lot of time and energy wasting nonsense around.

Rhonald Blommestijn

back

(5) Michael asked:

What would happen if "we" as in the human race never existed?

---

Convergence is a biological term that means parallel independent evolutionary development. The eye for instance, has developed at least six times in different lifeforms. The nomadic ants in Africa are very similar to those in South America but they developed independently. It seems that if something makes great sense in terms of genetic survival it will come to life. We are the only species with our level of intelligence and culture, and we can only speculate how coincidental our existence is. But it is very possible that if we weren't around, something very similar would exist.

Rhonald Blommestijn

back

(6) Paul asked:

Intelligence interferes with the natural course... Why did intelligence evolve?

Is it possible that mankind has not yet totally discovered the true purpose of his intelligence?

---

I guess 'natural course' here means: everything that happens if there are no humans? And I guess 'true purpose' refers to the evolutionary mechanism that brought survival benefit to those that developed the several stages of intelligence.

Your question refers to a scientific field that has hardly been explored yet. But there are a lot of theories around. Most of them are fascinating but very speculative. Fossils of our direct ancestors are rare and the brain doesn't fossilize at all. Read the wonderful book 'Darwin's Dangerous idea' by Daniel Dennett. Or anything by Richard Dawkins.

Rhonald Blommestijn

back

(7) Bob asked:

Why, in a democracy which grants us virtual freedom, are we forced to receive an education?

---

The answer is that no democracy really does "grant us virtual freedom". Despite the wording that appears in whatever (virtual or actual) "Bill of Rights" may apply to your jurisdiction, a democracy does not really grant individual residents much freedom at all. The laws, rules, regulations, and outright foolish restrictions imposed by a democracy on the average citizen's "freedom" fills rows of bookshelves in small print. And the list grows by a shelf or two every year. Take a look at any lawyer's reference library — and you'll see just one small part of it. I remember seeing an estimate for the growth rate in Canada in the 1960's — it was on the order of 50000 pages per year. And surely that rate has increased in more recent times.

The fact that you think your particular form of democracy does indeed grant you a degree of freedom is a testament to the social propaganda that reigned during your formative years. The "public image" of a democracy is one that stresses the rights of the individual to do as s/he pleases. The suppressed reality of a democracy is that the individual is allowed by everyone else to do as s/he pleases, so long as whatever that is does not impinge on the demand of that alien majority to be unencumbered by the consequences. The net result of this conflicting constraint is that your "freedom" is strictly limited by the ways in which the majority of your fellow citizens feel they might be inconvenienced by whatever it is you might wish to do.

The maximization of personal freedom is not the proper purpose of a democratic form of governance. The proper purpose of a democracy is the maximization of the welfare of the citizenry through the cooperative efforts of those citizens towards mutually desirable goals. Recent history (since 1850, say) clearly demonstrates that central management of such efforts is not nearly as effective as local and personal management of such efforts. But, if your particular interests in "freedom" are to pursue non-cooperative efforts, or pursue goals that conflict with the mutually desirable goals of the majority, you will find your freedom severely limited. Any so called "Bill of Rights" that might supposedly constrain the majority in this regard, will not really constrain them at all.

Which brings us to your actual question about forced education. The majority has decided, for various reasons, that forcing you to get an education is in the best interests of the majority in their pursuit of their cooperative efforts towards their mutually desirable goals. Some of the rationales that have been offered for this decision include:

(a) modern society is both socially and technically sophisticated, and if you are going to contribute to society and not become a parasite on the majority, you will need a rather sophisticated education. You will need to learn a great deal that you cannot easily learn any other way, if you are to obtain a productive means to self-sustainment, and if you are to avoid falling foul of one of those myriad ways of pissing off the majority of your fellows, and if you are going to raise up your own children in a "proper" (ie. acceptable in the eyes of that majority) way.

(b) A democracy works most effectively when, to the greatest possible extent, every citizen firmly believes that it is the best way to ensure their own personal welfare. And the education system has proved most effective at suitably "socializing" the young into this convenient belief.

(c) Young people are in the poorest of positions to appreciate the importance of these two rationales — they neither know enough yet (of history, sociology, and various technical fields), nor have been sufficiently "socialized" yet (to "properly" value things), to appreciate the longer range consequences of not having a "good" (ie. suitable to the interests of the majority) education.

Stuart Burns

back

(8) AskPhilosophers asked:

Is faith in something intangible ultimately delusional?

---

If it is, it is a delusion to believe that the internal angles of a triangle always add up to 180 degrees.

David Robjant

back

(9) AskPhilosophers asked:

If we built a computer that could analyse our minds, and it figured out how they work and explained it to us, would we be able to understand?

---

Well, how would it explain it to us? Binary code would be a tad difficult to follow. A several thousand word tract full of vivid imagery on every area of human life might be easier to follow, but the inevitable ambiguities would probably result in several thousand years of schism and conflict.

David Robjant

back

(10) AskPhilosophers asked:

Is it true that time has no end?

---

This isn't a scientific question but a grammatical one. What ends is in time. Time cannot be in time, any more than my hat can be inside my hat.

David Robjant

back

(11) AskPhilosophers asked:

1. What is a question?

2. Are there sentences that have the grammatical form of a legitimate question, yet nevertheless fail to be legitimate questions?

3. Does this sentence (i.e., (3)) have the grammatical form of a legitimate question, yet fail to be a real question?

---

(1) Just that. (2) Yes, for instance "Can you be serious? ", which in certain cases isn't an inquiry but an accusation. (3) Depends on whether you care about the answer in the way that someone who asks "can you be serious? " does not. Possibly you are just trying to be clever. But if said with the appropriate genuine curiosity, I suppose I would answer "No, it's a real question". All this ought to draw our attention to the fact that whether something is a question is more than a matter of "grammatical form", if by that we mean something about the words simply.

David Robjant

back

(12) Paul asked:

Thanksgiving day. I am thankful, but to who? My wife thinks that you can be thankful for, but don't necessarily have to be thankful to. I am thinking that if you are to be grateful or give thanks for something then that implies that you are grateful to something or someone. If I was thankful to no-one then I wouldn't be thankful.

Do you agree?

---

There is a saying, 'Thank your lucky stars.' You cross the road without looking and are narrowly missed by a twenty ton truck. You thank your lucky stars. Even an atheist can do this. You don't have to believe in the existence of such entities as 'lucky stars'.

But isn't the feeling I have just described merely a feeling of relief? What is the difference between that and a genuine feeling of thankfulness?

I can see the point of your objection. But I'm inclined to agree with your wife. The emotion of thankfulness has a formal object, but that object does not have to be an individual, it does not have to be someone I give thanks to. The most appropriate description for that formal object would be 'good fortune'. 'I am thankful for my good fortune', means, I appreciate my good fortune for what it is. (Which is not to imply any belief that my 'good fortune' is somehow written in the stars. The near miss was just blind contingency, nothing more.)

Not everyone appreciates their good fortune. Not everyone feels 'thankful' when they should. More's the pity.

Geoffrey Klempner

back

(13) AskPhilosophers asked:

At like an atomic level, like really small, is it possible to determine where one thing stops and another begins? Say like where my finger stops and a key on my keyboard begins? (This might be a bad example, because a plastic key and my finger probably have quite different atoms, but still the line between them would be hard to find right.)

---

It's a fine example, but matters are certainly complicated by your talk of atoms. Atom means in greek the uncuttable, and one of the original reasons for positing such objects was to solve worries like yours about where something stops. A thing ends, atomism holds, at the edge of it's last uncuttable. But nowadays atomic theory is pretty thoroughly cut up and we no longer speak of atoms as the minimum units of the world (there are discovered to be smaller bits and we do not theorise that we will find a smallest), whence your worries re-emerge. And rightly so. The discomforting thought is that there is an element of provisionality, of that-will-do-for- present-purposes, about what we call 'precision' here.

Atomism was originally meant to comfort us with the thought that our observations were an approximation to something which itself is very precise. Now we have to somehow cope with the idea of various degrees of precision, but without the idea that these degrees of precision are approximations to anything. At this point we may remind ourselves of the oddness of the question 'what time is it exactly'. We can say what time it is to the nearest second if we choose, or millisecond even. But since no unit of time is uncuttable, such measurements are only relatively exact, ie exact as to to seconds or milliseconds, and not exact altogether in the way that 2 plus 2 is exactly four. Here again we seem to have approximation without a thing approximated to — a grammatical nonsense. But the better thing to say is: 'when we say "approximate" we really mean "this will do for present purposes" and not "this is nearing final truth"'. The like remark must be made for your finger and keyboard. And for most purposes (establishing the correct angle of the keyboard for typing? ) quite large units of measure will do, like millimeters. What you are imagining for yourself is the quite odd and almost purposeless purpose of proving the point at which something really ends, which is perhaps philosophy rather than measurement — perhaps that is the explanation of why no measuring system could do the job you ask of it here.

David Robjant

back

(14) AskPhilosophers asked: Can you disprove the statement, 'Truth is relative'?

---

Yes. By directing you to the dialogue, the Theaetetus, where Plato does just that. What Plato allows is that Truth could be relative, if only the given world were made up of me, plus the little bits of stuff that I relate to, such that the relations between me and the bits of stuff are different from the relations between you and the bits of stuff. But the world isn't that way (the world is indeterminate and we have to intellectualize before we get any little bits or even any 'me'). What exists (flux) does not allow for the possibility that truth is relative, or in jargon, what Plato has done here is to reject the ontological assumptions of relativism. This differs from other 'disproofs' of relativism in the way that uprooting a weed differs from pruning it's sickly leaves. The 'no-self' view in Buddhism would perform the exact same trick.

David Robjant

back

(15) Josh asked:

I have to do a paper for english comp 1 (it's extra credit so I don't have to do it but I'd like to) Our teacher asked us why do good things happen to bad people and bad things happen to good people. He wants us to write a two page paper on it concentrating on something major in history (9/11, conquests, flu epidemics, black plague). Basically anything major in history that has happened to good people. any ideas? What is a good person anyhow, I for one don't believe anyone is good, but I told him that and he wasn't so "approving" of me writing a paper stating that. Any ideas or links you have about this would be much appreciated. One more thing... I'm not interested in a religious standpoint, I know why christians think it happens, but that's not something I'm interested in writing or even considering.

---

Try this approach:

Things that happen to people can be generally classed as "good things" or "bad things" according to some unspecified standard of evaluation. People also can be generally classed as "good people" or "bad people" by a similar (but not necessarily the same) unspecified standard of evaluation. Specifying the standards of evaluation involved in these classifications is not necessary.

Now, assume as a trial hypothesis that whatever happens, is in some sense a random event. What I mean by this is that it is generally unpredictable what things will happen to which people. If that is the case, then it will be inevitable that the things that happen to people (whether "good people" or "bad people") will average out to 50% "good things" and 50% "bad things".

Empirical observation of what does in fact happen to people would strongly support two conclusions. First, that people generally expect "good things" to happen to "good people" (and "bad things: to happen to "bad people") and therefore dismiss such occurrences as "normally expected performance". The opposite occurrences ("bad things" happening to "good people" and "good things" happening to "bad people") are noticeable and memorable because they do not fit into our expectations. Hence there is an observational bias towards the "bad things" that happen to "good people" (and the "good things" that happen to "bad people").

And secondly, it is empirically obvious that despite corrections for the above described bias, the distribution of "things" that happen to people does not seem to be balanced at 50-50 ("good" versus "bad"). It would appear that whatever qualities constitute being a "good person", they do in fact provide some insulation against a random distribution of "good things" and "bad things" happening. But however effective being a "good person" seems to be at tilting the distribution in a "good" way, enough "bad things" happen to "good people", that one cannot conclude that being a "good person" is totally effective.

Hence the conclusion that while the distribution of the occurrence of "good things" and "bad things" is not entirely random, it is not entirely determined by the "goodness" or "badness" of people. So we can conclude, in the words of Louis Pasteur "Chance favours the prepared mind."

Stuart Burns

back

(16) Christophe asked:

"There is a point I do not understand about final causes applicability to physical systems.

Action guided by final causes (future conditions as cause of the event) are not considered as scientifically acceptable in the world of matter. Only actions guided by efficient causes (prior conditions as cause of the event) are considered as scientifically acceptable in the physical world.

However, simple robots like the cybernetic turtle orientating its movement in the direction of a source of light can have their action guided by a final cause (reach the source of light). So a simple robot built up with matter following only efficient causes seems to be in a position to obey final causes. How is this possible?

---

In 'The Intentional Stance' Daniel C. Dennett distinguished between what he refers to as the "intentional stance" and what he refers to as the "physical stance". These are two different ways of viewing and thinking about a physical system. Most of science deals with the world from the "physical stance". As such, only physical causation — what Aristotle called "efficient causes" — is relevant.

However, we can also view the same physical system from the "intentional stance" and bring teleology (goals, Aristotle's "final causes") into the picture. The intentional stance views the system as an agent pursuing goals, and can understand the system from that perspective. Dennett's example was of a thermostat "wanting" to keep the room at a preset temperature. The example you provide is the simple robot turtle "wanting" to reach the source of light. At the physical level, however, all is "efficient cause". Photons from the light source impinge on a photo receptor, and through appropriate circuitry, "cause" the robot to move in that direction. Physically, all is "caused" by photons impinging on photo receptors and electrons following an electronic circuit from one pole of a battery to another.

The "kicker" is that the electronic circuitry has to be designed in the proper way to get the robot to behave as if it "wants" to reach the light. In the case of the robot, the designer was a human engineer. In the case of a real turtle reaching for some lettuce, the "designer" was the trail and error consequences of natural selection.

Stuart Burns

back

(17) Chris asked:

If you are holding all the feathers you can hold, can you hold one more?

---

Yes! Just stick it between my teeth. Oh no I've dropped it. Sorry!

Shaun Williamson

back

(18) Rich asked:

If I did not want to attend jury service what would be my best excuse.

---

Just say 'I'm an anti-social psychopath and I don't care what happens to anybody else as long as I'm ok. And I really don't have time to waste on this trivial rubbish'. That should get you out of it.

Shaun Williamson

back

(19) Ladymeth asked:

How can you answer the questions "Does life/ the universe have a purpose? " It seems to me there are too many ways to answer the question. If you are religious you may have a religious take on the answer. If you are a scholar you may have a scientific explanation. How do you even begin to approach an answer?

---

You are quite right that there are too many possible way of answering this question; and of course there is always the option of refusing. Or you may, as Bertrand Russell did in an interview, claim that there is no purpose beyond the 'raw fact' that the universe exists. However, I will suggest to you that there is an answer all the same. It is the answer which motivates us, and always has in the past, to keep looking and thinking in spite of the seeming hopelessness of this quest. That answer is: 'There has to be a sufficient reason for everything that is the case.' Now with this you have at least a tiny something in your hands with which to begin an approach. Of course, you might respond (or perhaps your friends might): why? what's to say there is any reason whatever in the universe? Well, that's not good enough. The point is, that we haven't discovered anything yet (ever!) for which we did not also find a sufficient reason why it exists or why it is just how it is. So anyone who wishes to doubt this has to come up with proof that it's all wrong.

Jurgen Lawrenz

back

(20) Kim asked:

Suppose that it was technologically possible to produce a sort of "Star Trek transportation device". Would the same person exit the device as entered? Would you get in one?

---

There are two plausible theories about how a Star Trek transporter might work. The first theory, which seems to be the most widely accepted version, is that when you get into the transporter booth, the atoms and molecules which compose your body (and uniform, of course) are disassembled and converted into some form of energy which is transmitted to the destination, where you are re-assembled. Given what we know about the equivalence of mass and energy, the transporter beam would be sufficiently powerful to annihilate anything in its way, but maybe the transporter exploits some kind of 'wormhole' effect.

On the second theory, when you get into the transporter booth you receive a 'total body scan' and the information is then transmitted by radio to the destination where it is used as the blueprint for recreating a version of 'you'. This seems a much less plausible account of what happens in Star Trek for two reasons. First, there is no mention of how your original body is disposed of (as it must be, to save the embarrassment of having two versions of 'you') nor of how the materials are obtained for building a body (with uniform) at the destination point.

Intuitively, we are more inclined to agree to be transported using the first method than using the second. But why is this? Because the individual who 'arrives' at the destination is made of the same matter? But why is that important? Suppose your 'matter' is beamed through space, but reassembled any old how, so that a carbon atom which was previously in your chin is now in your left toe. One carbon atom is as good as another. But then, if that's OK, what would be so wrong if some of the material was lost along the way and had to be replenished with carbon from a jar of 'people stuff', so long as the information was preserved?

You can see where this is going. Initially, there seemed to be a world of difference between the two methods of 'transport', but the closer you look at it, the less significant the difference becomes.

On either theory, would I get into the device? Think of it this way. Suppose I die suddenly, and at the very next moment, a perfect replica of me comes into existence at the other end of the galaxy — by pure, cosmic accident. The replica will 'think he is me', that is to say, he will remember writing an answer to Kim's question and so on. But so far as I am concerned that is a matter of complete indifference. I will die. End of story. It is no comfort at all to think that somewhere far away someone who thinks he is me will exist in my place.

But, then, what is it to 'survive' or 'not survive' from one moment to the next? Someone remembers starting to write this question ten minutes ago. By the normal, everyday criteria for personal identity no-one would doubt that that someone is the same someone who is writing these words now, namely me. Yet, so far as I am concerned, it wouldn't make any difference if I had come into existence ten seconds ago as a result of a cosmic accident, while the individual who started on this question ten minutes ago was annihilated.

I don't believe in a Cartesian 'soul substance'. Nor do I believe in a non-Humean causal link that mysteriously preserves, or fails to preserve identity over time. There is only the material GK who refers to himself as 'I' at this moment. If you can follow this reasoning, you will see that any form of Star Trek transport is equivalent to death, but that does not matter because as these examples show, there is no real difference between 'death' and 'survival' anyway.

Geoffrey Klempner

back

(21) Rosie asked:

Does 'a God who intervenes miraculously cannot be benevolent? ' logically make sense?

---

Yes, it makes perfect sense! Consider this: We are agreeing (I hope) that the concept of God which you propose is that of an omnipotent, omniscient spirit being — in a word the God of Christendom, Islam and Judaism. Now for this God time is a meaningless notion: time only exists for us so as (in the words of the old wisecrack) to prevent everything happening at once. But for god it does, of course. All eternity is for him just a picture, like you looking at a complete road map of your city. Now the important thing in this scenario is this: if this God wished to create a universe, he would not actually have to commit himself until he was totally satisfied that this universe is ok exactly as it exists in his mind. His mind would, in other words, contain an infinity of possible universes, and since he is all-powerful and all-wise and has an instantaneous grasp of all possibilities in all possible universes, and since finally God would have to be the supreme instance of infallible reason, he would hardly go to all the trouble of creating a universe that was less than perfect. Not even you or I would do this. If we build or write or play, and if in doing this we have a complete view of what we want to do and what its purpose is, we would not deliberately produce something less good than we can make it.

Agreed? I think you will. But from all this it stands to reason that if God, after he decided on creating this universe, intervenes miraculously in its progress, something must have happened that he did not foresee. Or maybe he changed his mind? Well, whatever the case may be, by changing anything in this universe, even the flight path of a mosquito, everything else in the whole universe would eventually change, because everything in the universe must be in some way connected with everything else in it. So even such a tiny change would over the long haul (maybe even millions of years, but to God this is nothing) change the evolution of the universe from what he had planned into something very different. This means that God goofed! Or else, that God likes to play games. But then, if that's the case, he is not the God we think of when we say 'God'. This god would have to be something altogether different. And therefore, whatever this god might be, he could no longer be benevolent, even if he tried. Because a god can't benevolent if he is not all-powerful and all-knowing. He would at least some of the time, by accident, have to be malevolent. He would have to take some promises and put up with things going on that are not happening by design. — Well, I hope you can see now why the logic of benevolence in respect of God is not something to muck around with. But interfering with the universe is mucking around. Therefore God does not.

Jurgen Lawrenz

back

(22) Askphilosophers asked:

Why do the laws of morality and the laws of nature seem to be completely opposite one another? For example, most moral codes encourage monogamy while the theory of evolution states the strongest seed should be spread around.

---

What tosh. There is no such statement of the theory of evolution that "the strongest seed should be spread around". What there is is an observation of fact, that the strongest seed will spread around, and even there I am not sure how much is actually being said, since "strongest" is here only defined in terms of what spreads around, so that the actual meaning of the claim is: "seeds that spread around will spread around". What an observation! The idea that you could construct a moral imperative on the back of this tautology is just barmy (or, more exactly, that brand of barminess known as 'social darwinism'). Of course, there are a lot of barmy people in the world, eg Adolf Hitler. How barmy are you?

David Robjant

back

(23) Askphilosophers asked:

I understand points as entities with zero extension. (Is this correct? ) Yet infinitely many points are said to compose space. It seems like even infinitely many zeros could never add up to a finite non-zero value. So, what's up with points? If they don't have any extension, what are they?

As a follow up, does it make sense to think about points in space in a different way from how we think about points in time?

To which there was entered the following response:

'Yes, a point has length, depth, and height zero. So do two points, three points, and even as many points as there are natural numbers. But if you have as many points as there are real numbers (of which there are more than there are natural numbers), then that set of points may have some positive length, depth, or height, though it may not. (In that case, they will not have zero length, depth, and height but may have no assignable length, depth, or height.) The branch of mathematics in which such things are studied is called "measure theory".

'Exactly what a point is is another question. In mathematics, points may be regarded in a wide variety of ways, as is convenient. Are there any points in space itself? That's a disputed question, and an empirical one, not one on which philosophers can pronounce.'

---

This response is a response to the wrong question, and thus confuses matters as only a philosopher can. What the questioner had asked was whether infinitely many points could compose space. What the answer says is that "if you have as many points as there are real numbers [...]" then the set of points may be supposed to have some extension. But that it is possible to suppose some unassignable extension to a set of points says nothing about whether points compose space. The answer here is an answer to a question that the questioner didn't ask, namely whether is was possible to suppose that sets of points could be extended. What the questioner wants to know is whether space is composed, i.e. made up of, a set of points. And on this the questioner's intuition is completely correct, namely in this, that several nothings do not add up to something.

I may add that the expression "as many points as there are real numbers" is peculiar. It looks like it is specifying a quantity. It does no such thing. For instance, it is logically impossible that there be as many apples as there are real numbers.

David Robjant

back

(24) Askphilosophers asked:

When the 1st word was spoken (i.e., Oog the caveman says "rock"), could it have been really understood without a language in place? Could Oog have really understood he was saying rock unless he already had the system in place. In other words would Oog have to have a word for "I", "am", "going", "to", "name", and "this", etc.? In other words would not the 1st intelligent being to speak something intelligible have to have a basic language in place? If this is true, then what would be the ramifications of that? Maybe the question should be posed, "Is it possible to think without a language? "

---

Sensing things, like trees and rocks and footfalls in the forest, as invested with all kinds of further significance for the future, is so central to life even for the most basic mammal, that I find it hard to understand what the big deal is with the honorific "the first word". To the tiger as to the man, the leaves and tracks and sounds of life are already signs, already words in that sense. That some naturally uttered sounds might over time come to have more complex or more distinct signification for human beings (or for that matter tigers) does not seem to me to pose any great difficulty.

What tends to happen, in order to provoke some difficulty, is that questioners speak grandly of "language" as if is this referred neatly to everything currently given in the Shorter Oxford Dictionary (and not to the strange hrumphing and cooing noises that your partner makes while watching TV), and then get themselves a neat self-made puzzle about how all that stuff in the Dictionary first got going. Not from nothing, obviously, and not from 'Aardvark' either. Well the error is in supposing that language is a neat contained object on this model. Tigers think, and Humans think, and sometimes they think about footfalls just in the way that some of them think about essays and questions (what kind of animal was that? ).

David Robjant

back

(25) Askphilosophers asked:

How do formal logicians respond to Marxist/ Leninist/ Dialectical logic claims? For example, in "An Introduction to the Logic of Marxism", George Novack explains that the law of identity of formal logic, that "A is equal to A", is always falsified when we try to apply it to reality. Here is a quote from the book, in which he quotes from "In Defense of Marxism" (it is long, I apologize):

"... a pound of sugar is equal to itself. Neither is this true — all bodies change uninterruptedly in size, weight, color, etc. They are never equal to themselves. A sophist will respond that a pound of sugar is equal to itself at 'any given moment.'

"Aside from the extremely dubious practical value of this 'axiom, ' it does not withstand theoretical criticism either. How should we really conceive the word 'moment'? If it is an infinitesimal interval of time, then a pound of sugar is subjected during the course of that 'moment' to inevitable changes. Or is the 'moment' a purely mathematical abstraction, that is, a zero of time? But everything exists in time; ... time is consequently a fundamental element of existence. Thus the axiom 'A is equal to A' signifies that a thing is equal to itself if it does not change, that is, if it does not exist."

He goes on to say that since everything exists in time, the law of identity of formal logic is never applicable to the real world. Presumably at least one reason we use logic is to help us arrive at beliefs that will then guide our actions, so presumably if the law of identity of formal logic were in fact falsified when applied to the real world, that would be a problem. What do you make of all of this? ?

---

I think it is the least stupid thing I've ever heard described as "Marxist/ Leninist/ Dialectical logic", though I wonder about the accuracy of the description and might be getting a bit lost about who is quoting who in the above. How anyone squares an awareness of the limitations of intellectualisations in the face of flowing reality with a belief in the soundness of Dialectical Materialism, I just wouldn't know. If anyone does do this, then either it indicates that you can't fool yourself all the time, or alternatively that if you are really good at fooling yourself any combination of incompatibles will do.

David Robjant

back

(26) Tomm asked:

What significant impact has the field of philosophy had on society as we see it today?

---

We notice what we rely on most, like the central heating boiler, when it fails us. And philosophy's catastrophic failures (or rather our failures to think well in the way that an awareness of the history of philosophy can inform) have had too many significant impacts on society to mention them all. For an absolute classic case, the rise of Hitler, and the daft social darwinism at the heart of Nazism, would be hard to beat.

David Robjant

back

(27) Peter asked:

I'm interested in understanding why rule-following has been so central to the philosophy of mind and language?

---

Well, what's a "rule"? It's a way to formalize regularities, right? So whenever you have regularities, the idea is that you should have rules, i.e., ways of describing those regularities in general terms. Now, what's a language? Well, it's a symbolism which we learn in order to communicate, let's say. But what makes learning it possible? If words, etc., had no regularities, then there would be no point in trying to learn them, right, because without regularities, they'd happen randomly. But they don't, we learn them... etc. So, the reasoning goes, we must be utilizing rules, in some sense, even if we don't know what those rules are, because our being able to speak coherently implies that we can reproduce the regularities in the language we speak.

You get it? Now, apply the same argument to mind, i.e., to thinking. The assumption here is that regularities imply rules. An interesting assumption, which can be questioned. See, e.g.,

Dreyfus, H. L. "The Current Relevance of Merleau-Ponty's Phenomenology of Embodiment." Electronic Journal of Analytic Philosophy 4 (1996).

------. "Samuel Todes's Account of Non-Conceptual Perceptual Knowledge and Its Relation to Thought." Ratio XV, no. 4 (2003): 393-409.

Johnson, M. "Imagination in Moral Judgment." Philosophy and Phenomenological Research 46, no. 2 (1985): 265-80.

------. The Body in the Mind. Chicago, IL: University of Chicago Press, 1987.

Steven Ravett Brown

back

(28) Busy asked:

A person says they will marry only race A. This is discrimination by race. By definition this is racism. Does one have to listen to the reasons of the first statement before they can conclude the definition? In this degree is racism still unethical?

---

No, its not clear that this is racism, it might just be preference. Some people only want to marry people who have blond hair. Some people only want to marry people who are taller than them. Some people only want to marry somebody who is rich. It all depends on their motives. Are they just following personal preference or are they motivated by an irrational hatred of other races.

In Britain for example black people and Asian people don't want white people to love them or to marry them they just want to be treated the same as anyone else with regard to employment, housing, education etc.. They don't want to be the target of irrational hate.

When it comes to marriage then like your choice of ice cream you can just follow your own personal preference. However if your preference is based on irrational hatred of other races then you are a racist.

Shaun Williamson

back

(29) Bob asked:

From a philosophical point of view, how can you best describe an image of 'nothing'? It's part of a college project I'm working on.

---

The best way to approach this would be to stop thinking in terms of images. As "nothing" is an absence of anything (including images) there is no image that would suffice. "Nothing" is a concept and cannot be visualized.

Kevin Macnish

back

(30) Scott asked:

Excuse the structure of this question. Assuming God is omniscient, omnipotent, and omnibenevolent right? Then you have Augustine's evil as a privation of the good theory coupled with the Trinity theory where any creation of God can not be supremely and equally and unchangeably good, but are good nonetheless right? Then you have the fact that God created Heaven. Now this means that Heaven is not perfect and therefore susceptible to corruption, according to Augustine. But the Bible states that there is no suffering in Heaven.

Now, if God is all knowing, and he knows when you are thinking of sinning but because of the free-will argument, allows it, what kind of people does God let into Heaven? Surely everyone has a thought of sin, and so to what extent does God allow tickets to heaven. I fail to see how Heaven can be free of suffering if everyone is liable to corruption, and if God does remove the thought of evil from the people in heavens mind, then the free-will is gone, and the existence of humans is downgraded. So I fail to see how there can be a heaven.

Another point: Angels? What's the go with them? Are they creations of God? If so does that make them liable to corruption and imperfect. There's so many contradictions in the Bible that I don't see how anyone can possibly be a believer.Noah's ark is a perfect example of this. "Ok the world is becoming full of evil, why don't I get someone "good", put them on a boat with other species and delete everything else and start again". Why doesn't He do that now? or is the world not full of evil? Please correct me if I'm way off the mark here, and sorry if I have wasted your time but I was just curious.

---

No need to apologize — these are all good questions. As regards your first, that Heaven is susceptible to corruption and yet there is no suffering in Heaven, I believe that the bible says that there will be no more suffering in heaven. That is, it is a future promise rather than a current description. Given the historical orthodox understanding of the fall of Lucifer there clearly has been suffering of some sort in Heaven.

Secondly, yes, everyone has thought of sin and by some of Jesus' strictures on the Sermon on the Mount, this is sin. However, God does not allow people into Heaven based on whether or not they have sinned. If so then it would be one empty place. Rather, again, the traditional orthodox belief is that one goes to Heaven as a result of belief in Jesus.

Thirdly, once in Heaven one comes face to face with God, and remains in his presence for an eternity. On encountering him directly the theory goes that his goodness is so overwhelming that one will no longer desire to sin, although he/she would remain free to do so.

Fourthly, yes angels are creations of God (all things are except God himself) and corruptible, hence the account of the fall of Lucifer.

Fifthly, maybe the world is not as full of evil now as it was in Noah's time — who knows? ! This is by no means a waste of time, but you would do well to read a decent systematic theology (e.g. by Norman Geisler or Wayne Grudem) and a book that deals with apparent and real contradictions in the bible (again, "When Critics Ask" by Norman Geisler and Thomas Howe would be a good start). These would help you grapple with these questions more thoroughly.

Kevin Macnish

back

(31) Nina asked:

How did Descartes try to prove God's existence?

---

Descartes employed a form of the ontological argument, first used (to our knowledge) by Anselm of Canterbury. This essentially says that God by definition must be perfect in every way. Yet existence is a perfection. Therefore God exists. He also used a form of the cosmological argument, starting from his position of doubting everything except his own existence. He reasoned that if he doubted then he must be imperfect (for he lacked something — i.e. knowledge). But to know that one is imperfect implies that one knows of perfection. This concept of perfection cannot come from an imperfect being.Hence there must be a perfect being (God) from whom the concept of perfection is derived. Both of these are discussed in his Meditations.

Kevin Macnish

back

(32) Bill asked:

Is there any connection between language and math? Are equations sentences? Both can be true or false. Does it matter?

---

In maths, some equations are sentences, i.e. state something which is true or false. For example the arithmetical statement 5 + 7 = 12, or a general formula such as (a + b) x c = (a x c) + (b x c). However, not all strings of symbols which have the form of an equation are statements with a truth value, for example y = 2x, which describes the function 'times 2'.

It matters a great deal to philosophers of mathematics how we understand the nature of 'truth' in mathematics, and how this relates to our understanding of truth as applied to contingent statements about the world. I guess that this is what your question is driving at. Are numbers 'objects' existing in mathematical reality (whatever that is) or merely marks on paper or ideas in our heads? That is one way, a not very accurate way, of posing the problem of truth in mathematics. Another way to pose the problem would be to ask whether it is conceivable that there are mathematical truths which cannot — even in principle, given unlimited amount of time— be proved.

Look up: Intuitionism, Platonism, Formalism. One book that you might look at is Putnam and Benacerraf Eds. 'Readings in the Philosophy of Mathematics' CUP 2nd Edn 1983.

Geoffrey Klempner

back

(33) Chris asked:

What are the implications of accepting each of the views identified by Socrates (that God came before the good, or that the good came before god) why might people find these ramifications problematic? Could god create a statement that is both true and false? Did god create the rules of logic or is he subservient to them? And if so what are the ramifications?

---

The implication of accepting that God preceded the good is that what is termed "good" is only such because God calls it so. It is hence arbitrary and liable to change if God changes his mind as to what he thinks of as good. If, however, the good precedes God, then God is subject to the good and hence not the supreme being. The same could be said of the rules of logic. The orthodox Christian response is that God is good, and God is logical. That is, it is his nature to be good and his nature to be logical. Hence no, God could not create a statement that is both true and false.

Kevin Macnish

back

(34) Lee asked:

Assuming for sake of argument that God is the original energy source that created the universe, then what materials did he use to create with? If only God alone existed then wouldn't it stand to reason that the only materials he had in order to create with was his own essence, energy etc.? Would this mean that pantheism or monism is a more logical form of theism? Otherwise you must posit creation ex nihilo which simply seems to me to be an appeal to authority at best and an appeal to magic at its worst.

---

Pantheism/ monism believes that god and the universe are one throughout eternity, and so there was no creation: god/ the universe have always been here. Theism, though, does hold that god created the universe and that he did so ex nihilo. I'm not quite sure how this is an appeal to magic, although I can't think of anyone who would deny that, if it happened, it was miraculous. Nor is it an appeal to authority. Logically, the universe either had a beginning or it did not. If it did (and that is what we are assuming here) then it must either have happened ex nihilo or out of pre-existing matter. But where did that pre-existing matter come from? That leads us down an infinite regress and is tantamount to saying that the universe is infinite. Hence the choice would appear to be a stark one: either the universe is infinite or it was created ex nihilo.

Kevin Macnish

back

(35) Erica asked:

Can you explain why the proletariat class wanted to destroy the bourgeoisie class? Also what is the meaning of the Communist Manifesto? What does it mean by abolition of private property?

---

Marx believed history demonstrated that there was a persistent, inevitable struggle between the classes for supremacy. This was furthered by the fact that the ruling class tended to abuse the classes that were below it. During the feudal period this took the form of revolt against the aristocratic ruling class. Following the industrial revolution, however, the bourgeoisie rose to prominence and power, and so the focus of revolt shifted to the middle classes. In particular this took the form of the proletariat against the bourgeoisie as the latter used the working classes in their factories under extreme conditions which deprived them of their dignity. It was in seeking this dignity that the proletariat would rise up to overthrow their capitalist "masters". Private property was a mark of the ruling class and ultimately gained at some point in history through theft (war, etc.). Hence to say that land "belonged" to anyone was ridiculous. Rather it should be owned in common. That way property could be shared fairly from those who had most to give to those who had most need of it. The most efficient way to achieve common ownership (rather than private ownership) was for the state, the representative of the people, to own all property on behalf of the common good.

Kevin Macnish

back

(36) James asked:

My question relates to the role of "belief in the afterlife" in ethics. How significant a role does belief, or lack of belief in the afterlife, play in defining ethics? And specifically, from the perspective of someone who does not believe in the afterlife, would it be true to say that the belief of others in the afterlife is the most fundamental obstacle to agreeing on standards of ethics? i.e. Those who do and do not believe that there is another world where wrongs will be righted will always disagree with each other fundamentally on what is or is not ethical in this present world.

---

Both would almost certainly agree that murder is wrong, and so it would be wrong to say that each would always disagree with the other on ethical issues. Furthermore, a belief in the afterlife may have very little role in defining one's ethics, if one believes, for instance, that how one acts in the current world has no impact on where one ends up in the afterlife. What may be a more pertinent area of conflict would be between belief in a god and no belief in a god. In this case, the believer is more likely (though by no means forced) to take a deontological ethical position (i.e. defining something as right for the reason that god says that it is right). The non-believer, however, is more likely to take a teleological stance (i.e. defining something as right because of the impact that it will have). However, even these positions are not necessary, as non-believers may be deontologists and believers may be teleological.

Kevin Macnish

back

(37) Chimaobi Opara asked:

Please help me to examine the reasoning and revelation, compare and contrast the two as sources of the knowledge of divine truth, the truth of God.

---

This is a hard question as revelation describes the source of knowledge, while reasoning describes the methodology applied to that knowledge. There are generally acknowledged to be two forms of revelation: general and specific. General revelation concerns the way in which God has revealed himself in the world (such as the complexity of creation). Specific revelation concerns the way in which God has revealed himself in writings (such as the Bible, Qu'ran, etc.). Reason is then applied to each of these areas to derive knowledge of God.

Kevin Macnish

back

(38) Bob asked:

I have a question regarding the determination of whether a question is a "valid" question. I know that in set theory there are restrictions on how a set can be defined to avoid a paradox such as who shaves the barber in the town where he shaves everyone who doesn't shave themselves. Are there any such "rules" for whether a question is a valid question? I think we often tie ourselves in knots arguing about answers to questions that have no answer since they are in some sense invalid questions. Perhaps my question is one such question.

---

The short answer to your question is no, there cannot be a set or rules for determining if a question is valid. If a question is grammatically and semantically correct then it is a valid question but it may not be a sensible question. Also not all valid questions have an answer. Set theory is a branch of mathematics and mathematicians are entitled to agree on restrictions in any branch of mathematics in order to make it useful since a branch of mathematics that contains contradictions cannot be useful. A question can be valid without being sensible. Consider for example 'How many angels can dance on the head of a pin? '. Since angels have no physical body, no sort of measurement could answer that question. Also we need to consider the context or use of a question. For example the question 'Who am I?' is sensible when asked by someone suffering from amnesia and it has sensible answers. However when someone asks this question in philosophy then it has no context and it is not clear if it is sensible or how we are supposed to answer it.

In a similar way if someone says 'Can you prove that I exist?' then if we are in a legal context the answer is yes. There are legal standards for proving that someone exists e.g. a combination of the legal documentation and finding someone who witnessed your birth and knew you as a child etc. But again the same question asked in a philosophical context may not be sensible.

Shaun Williamson

back

(39) Chris asked:

How many licks does it take to get to the center of a lolly pop?

---

This is a scientific question not a philosophical one. It depends upon the diameter of the lolly, how rough your tongue is and the strength of the licks you apply to it. The flavour of the lolly would also have an effect on the result.

Shaun Williamson

back

(40) James asked:

Why is it that as many people get older they long for a place in the country or near fields or beaches, or small towns, and away from cities. Is it a re-identification with nature? Is it a romantic notion? is it a response to stresses? Why is this being seen in most western countries?

---

This is a psychological question not a philosophical one. There is a psychological trait known as 'Surgency' which is roughly the ability to be positively stimulated by novel social situations. Surgency is highest in adolescents and is probably connected with the sex drive. Younger people crave excitement. They like to go to crowded clubs with loud music. Young people need to meet lots of potential mates and they can do this better in large cities.

When people enter other stages of life such as rearing children then they may prefer a quieter more predictable environment and when they are old they may not feel the need to meet new people. However this is a very individual thing and some old people retain a high degree of surgency into old age while some young people have a low degree of surgency.

Shaun Williamson

back

(41) Carlotta asked:

What is Socrates' view on the corruption of the youth in the trial and death of Socrates?

---

In Plato's Dialogue 'Apology', Socrates argues that, what he only does is to go about persuading the citizens of Athens to care not for body and money but for the excellence of the soul, since all the human goods derive from virtue and not the contrary (Apology 30a). And when he is accused by Meletus that in his teachings he acknowledges no gods at all, Socrates argues that, is impossible that one teaches things that pertain to divinities and does not acknowledge that there are divinities (Apology 27c). Finally he argues, among the supposed 'corrupted' by him must be some who as they are grown old by now, they must have realized that he counseled them towards evil and would have come forward to accuse him. However, none of them or their relatives has come to accuse him, but on the contrary all of them have come to support him (Apology 33d-34b).

Nikolaos Bakalis

back

(42) Zach asked:

What is the most extensive, compendious catalogue of logical fallacies, both formal and informal (Irving Copi, 'Introduction to Logic', mentions one citing over two-hundred — is that the largest?), and in which books might I find this?

---

Go here:

http://skeptically.org/logicalthreads/

Steven Ravett Brown

back

(43) Askphilosophers asked:

How can we rationalize societal condoned killing like war and execution. Is our collective conscience so bereft of compassion that killing others in the cold light of day is ok, especially if our peers say it is?

---

War and execution are different case and should be treated differently.

It might be argued that it is precisely because of compassion that we sometimes find ourselves killing, or justifying killing. Something called 'compassion' which involved any number of pious remarks and standing idly by whilst those who you feel with or for get tortured/ threatened/ dehumanised/ dispossessed/ killed etc would be a very odd creature indeed — it is argued.

I have to say that despite some misgivings I feel the pull of that argument. The difficulty with it — a difficulty which intelligent pacifists are quick to identify — is that there is an apparent imponderable involved in 'not standing idly by'. A pacifist will point out that he does not stand idly by when faced with acts he objects to, but protests, actively, sometimes with success. He will say: 'Is this not action enough to give content to compassion? Why must the kind of action necessary to give content to compassion include, at the extreme, killing? ' And he has a point. No one could call Gandhi's campaign against the British 'standing idly by'. And this may often be used to give volume to the 'there must be some other way' complaint against those whom pacifists are pleased to call 'war mongers'.

However, and here's the rub, there are enemies against whom all resistance but military resistance will be fruitless, and will result in the victory of that evil. To be aware of this fact, and not fight in those cases, would seem to me a clear a case of standing idly by as can be had. And that, I maintain, would hardly be compassion. This is, as I recall, the argument George Orwell had with pacifists in the 30's and 40's.

It is worth adding that to justify war against otherwise invincible evil is not of itself to justify all means in war. There is a much derided but for all that quite glaring difference between actions directed at stopping a combatant by killing him (justifiable), and specifically setting out to kill a civilian (unjustifiable). (To my mind that suggests an important difference between killing combatants in warfare and judicial murder, since in the latter case the victim is already, in virtue of being in a court situation, out of combat. Sometimes by 'execution', however, you mean being shot by a police officer who quite reasonably wishes to defend himself or the public in an emergency situation — and that would be more like war.)

By the way, what our peers say is OK certainly ought not to have much of a role in our deciding what is right. Unfortunately, it is precisely because peer pressure does in fact often have this role that great armed evils can arise (Hitler, Stalin, Saddam, Bin Laden etc), against which nothing but appropriately targeted military action can avail. So if someone were to say "the 7/7 terrorists went through the same process of peer-pressure and reinforcement that makes killing possible for the British army, so they are just the same", they will have said something with a true premise, and a patent non-sequitur. That all alike go through a process of 'societal condoning' to some degree is true, but does not establish that all societies are equally right in what they happen to condone. That society has a role in making military killing possible says nothing whatsoever about whether a particular killing is right or wrong.

David Robjant

back

(44) Askphilosophers asked:

Is time stationary, and we move along it? Or are we stationary, and time moves past us?

---

Metaphors of movement have their uses here, but they do not, I think, include speculation about direction. I suggest you look up William James on the pseudo question of whether the rabbit runs around the tree, or the tree around the rabbit. Practically speaking, your two scenarios would not differ in experimental terms. They would only differ in that the idea that time is a stationary landscape against which we move already contains something called the 'block universe' view, an article of blind faith that the way the world will be is in some sense already arranged, which has consequences for free will. It seems to me that to chose between two practically and experimentally indistinguishable hypotheses with the aim of arriving at that one which has the most disastrous consequences for our ordinary practice is... just bizarre. A better option would be to opt for neither hypothesis, reminding ourselves that the idea of time as a kind of movement along a line or a dimension is a metaphor, and compels no speculations about relative direction.

The one true usefulness of movement as a metaphor for time here is for making the important point that time as we experience it is one particular kind of movement if it is any movement at all, namely the smooth continuous kind — hence Einstein and his trains, Heraclitus and his rivers, Zeno and his Arrows, and so on.

David Robjant

back

(45) Elayne asked:

Please explain the Heraclitean-Parmenidean dilemma in laymen terminology. Thanks.

Sandy asked:

How do Parmenides & Heraclitus conclusions over the question, "What is the ultimate non-physical force that unifies and governs all things?" differ from one another?

---

Heraclitus:

There is a universal law of eternal change through the strife (polemos) of the opposites (enantia: contrary qualities), the so-called 'fiery' Logos. These contrarieties can be transformed into one another, and when they are in balance appear as 'one' (en). Following the Ionian School, in order to explain the transformation of the physical elements one into another, Heraclitus introduced the universal stuff of 'fire' (pyr), which is common in all things, just like Thales' 'water' or Anaximenes' 'air'. However 'fire' is nothing but the 'energy coin' in all changes, just like the transformation of mass into energy and the contrary. 'All things are an equal exchange for fire and fire for all things as goods are for gold and gold for goods' (Fr. 90 Diels).

The metaphysical principle of 'sufficient reason' for the coming to be and passing away of Anaximander (justice, penalty and retribution) is for Heraclitus 'privation' (chrismosene) and 'satiety' (koros), and is determined by the universal constitution 'Logos'. 'All things are in flux' in the phenomenal physical world and nothing stays still, and the only common entity in all things is the universal 'Logos'. Logos possesses prudence, acts in accordance with Justice and expediency — even the very sun must comply with it — and is sufficient for all. 'Of Logos, which is as I describe it, men always prove to be uncomprehending, both before they have heard it and when once they have heard it. For although all things happen according to this Logos, men are like people of no experience, even when they experience such words and deeds, as I explain, when I distinguish each thing according to its constitution and declare how it is.' (Fr. 1 Diels). Heraclitus'

Epistemology includes sense perception, interpretation of sense-data, as well as definition through privation and negation. 'They would not have known the name of justice, if those (abuses) there were not.' (Fr. 23). 'The things of which there is a seeing, and hearing and perception, these do I prefer.' (fr. 55).

Parmenides:

There is no eternal change; only the Being exists (einai) and lies behind the phenomenal world. All the material forms are different appearances of the Being. The Being is one, ungenerated, indivisible, continuous, imperishable, changeless and motionless, therefore Parmenides was called 'Stasiotes' (stasis: immobility). 'For you will not cut off for yourself what is from holding to what is, neither scattering everywhere in every way in order nor drawing together' (Fr. 4 Diels).

What really exists always in all things are the immaterial, imperishable and intelligible entities, which are attributes of the Being, and through those we can know what really is (estin). Therefore the knowledge of what is, can only be possible through reasoning and strife-encompassed refutation in order to discover these entities. The Forms of Plato were inspired by this notion of Parmenides. 'For the same thing exists for thinking and the being' (to gar auto noein estin te kai einai) (Fr. 3) 'But look at things which, though far off are securely present to the mind' (Fr.4) His hypothesis starts from the fact that, we cannot know things through negation (e.g. not-white, not-man etc., if we don't know the 'white' or the 'man' etc.), since only the being exists. Therefore one should focus on what really is and not on what is not. The being always exists it is immaterial, intelligible and knowable. The material apparent forms are passing, do not always exist, are not being, therefore unknowable.

'Come now and I will tell you — and you must carry my account away with you when you have heard it — the only ways of enquiry that exist for thinking; the one way, that it is and that is impossible for it not to be, is the path of Persuasion, for it attends upon the Truth; the other, that it is-not and needs must not-be, that I tell you is a path altogether unthinkable for you could not know that which is-not (that is impossible) nor utter it' (Fr.2).

Therefore, Parmenides is considered to be the initiator of deduction and the founder of Metaphysics.

Nikolaos Bakalis

back

(46) Daisy asked:

What makes something beautiful?

---

This is THE question of one of the relatively small but still enormous department of philosophy called 'Aesthetics'.

To a philosopher asking 'what is beauty? ', is almost similar to the questions 'what is good? ' or 'what is true?'. It's too big a question!

I suggest you start at this link: http://en.wikipedia.org/wiki/Aesthetics

Rhonald Blommestijn

back

(47) Sarah asked:

Hello, I was wondering if you could help me with this question?

Can belief in the soul be considered to be rational?

---

Yes, it can very much be considered to be rational. But it cannot be considered to be empirical.

Rhonald Blommestijn

back

(48) Jane asked:

Do we need god to become morally right?

---

Once religion was thought to be the only defence for any form of morality. But now it seems much more likely that the sense of right and wrong is something we are born with. Religion — or any other social context — can shape this, but it isn't the source.

There is no evidence that religious people think more morally than atheists do. There is a small statistic difference in behaviour but that can be emplaned by the fact that the believers are more often part of a social group.

Rhonald Blommestijn

back

(49) Natalie asked:

What are the differences between the Psychoanalytical approach and the Behaviourist approach.

---

They are different beyond compare.

There is very little about the psychoanalytical approach that can be considered scientific. And in a whole century PA hasn't produced anything that qualifies it as even remotely practical. Please consider Freud cs. as influential fictionalists.

The behaviorists are the opposite. They were most times very practical and scientific But they weren't persuasive enough to be widely influential. Not many people liked the idea of being compared with pigeons.

Rhonald Blommestijn

back

(50) Jorge asked:

I am currently studying Berkeley and his idea that nothing exists, I was wondering if you could clear something up for me. I figured out from his argument that the material world only 'exists' in our minds, but my concern is if it exists in our minds, shouldn't our minds exist, thus negating the argument that nothing exists? Now. if our minds are simply non-material things, then is Berkeley forced to subscribe to the idea that non-material things exist, but material things do not?

---

Your view that Berkeley is suggesting that nothing exists is misconceived. he did not say 'nothing exists', he produced reasons for saying that the immediate objects of our senses do not exist independently of us. So we ourselves, in the form of minds, must exist. The simple fallacy that you reveal is the common one of believing that when philosophers talk about reality or existence they only mean 'material' things. Berkeley is pointing out that we can consider reality in other ways. His notion is that the world consists of nothing other than minds and ideas, hence, to Berkeley reality/ existence is bound up in the mind, and he asks opponents to prove that the world is constructed in any other way. In his 'Three Dialogues between Hylas and Philonous' he undertakes to prove that there is no such thing as matter at all. there is no room to enter into the argument here, but those who support the philosophy of Idealism believe that Berkeley makes it very difficult for materialists to support their concept.

Arising from his argument is the notion that we are contained within our minds, the 'I' within the mind may be one way of looking at it. Berkeley's argument, as Russell points out, "[shows] that the existence of matter is capable of being denied without absurdity, and that if there are any things that exist independently of us they cannot be the immediate objects of our sensations." There is evidence within Berkeley's argument that he regards the notion of 'matter' as an invention to try to come to terms with the sensations we receive every second of our waking lives. Such a notion also entails the invention of 'space' and 'time' in which to locate such material objects, hence, that which is claimed to be 'external' is nothing more than a complex series of unsupported inventions, to prove the existence of a world external to the real world which exists in our minds by the grace of God. Though complex, and again too elaborate to be entered into here, Berkeley holds notions with regard to space and time.

Berkeley's argument forces us to choose between the reality of 'matter' or the reality which is obvious to us as 'ideas' in the mind, taking into consideration that matter is quite unknowable and can only be inferred, whereas ideas are presented to us as observable facts. The term 'idea' as used by Berkeley is rather different to the common, everyday use of the term. He applies 'idea' to anything which is immediately known, such as 'sense data'. Things remembered or imagined are ideas because when brought to mind we have immediate acquaintance with them. hence, only that which is perceived is real.

You may find the following helpful:

The Problems of Philosophy, Bertrand Russell, Oxford Paperbacks.

the British Empiricists, James Collins, The Bruce Publishing Company.

Berkeley Philosophical Works, Professors A. A. Luce and T. E. Jessop, Guernsey Press.

Berkeley, George Pitcher, Routledge & Kegan Paul.

John Brandon

back

(51) Christine asked:

Are there any credible links (or interesting parallels) between the metaphysics of 'possible worlds' discussed in modal logic and the multiple-dimensions suggested in the string theory?

---

There might be various reasons for postulating multiple dimensions or multiple universes in physics. You mention string theory, but a more well known application of the idea of possible worlds is in the many worlds interpretation quantum mechanics. Consider, for example, the click of a geiger counter, caused by the emission of an alpha particle from a lump of uranium 238. On the many worlds interpretation, the event of the particle being emitted at a given time 'creates' two universes, one in which the particle was emitted at that time and one in which the particle was not emitted. The result is a rapidly growing set of 'alternative' universes.

In modal logic, every statement which is not self-contradictory is true in some possible world. For example, the statement that Germany won the second world war, or the statement that Kennedy was not assassinated but died in his bed of a heart attack. Intuitively, there are many more modally possible worlds than there are 'multiple dimensions', or 'many worlds', or 'multiverses' or any such notion posited by a scientific theory. Modally possible worlds exist as a matter of logical necessity, whereas multiverses are 'created' only when certain contingent conditions obtain.

Our notion of the 'existence' of a multiverse in physical theory would be comparable to any other theoretical posit. As with atoms or gravity you can take a 'realist' or an 'instrumentalist' view. Similarly, in the semantics for modal logic, there are different views about the 'reality' of possible worlds. The considerations for and against might be comparable in the two cases, or they might not. There is no reason in principle why one might not a realist with regard to multiverses but a constructivist with regard to possible worlds, or, alternatively, an instrumentalist with regard to multiverses and a realist with regard to possible worlds.

Geoffrey Klempner

back

(52) Christine asked:

I have rather a lot of questions to ask about consciousness. Although I have read a couple of books to find the answers they seem to throw up even more questions. I will keep searching and trying to understand but I was hoping that Ask a Philosopher could help me in a succinct way.

1. What is a phenomenal concept and how do they differ from perception?

2. If a phenomenal concept is as I understand it, doesn't this put a physicalist in a shaky position?

3. What is functionalism? Is it possible to be a functionalist and a dualist (or a functionalist and a physicalist)?

4. I am a dualist. Not in the Cartesian sense where I believe the mind is quite distinct from the brain, I feel the two are very interdependent on each other. I also think that mental (non-material) thought can have causal effects on the material world (so I reject epiphenomenalism). What kind of dualist am I? Does that mean I'm an interactive dualist? Or have I gone wrong somewhere in thinking about the whole problem?

I'm sorry to ask so many questions that could easily be placed into a whole Blackwell Series. And I will continue my reading in the search for these answers. But if you could help in any way it will really ease the agony I am experiencing when thinking about the mind-body problem.

---

Well, I'm a physicalist, so I don't think you're going to like my answers. But, here goes...

1) I'm not sure what you mean by "concept" here. Are you asking about the difference between non-perceptual, abstract concepts and percepts? If so, there's a fair amount of controversy here as to whether there can be abstractions not ultimately derived from percepts (in the sense of apprehensions). You might look at this paper:

Goldstone, R.L., and L.W. Barsalou. "Reuniting Perception and Conception." Cognition 65 (1998): 231-62.

It's quite an interesting argument against there being a fundamental difference. The whole issue of "perception" is a very complicated one, because there's really no way, without extreme difficulty and very closely controlled laboratory conditions, to tease out what are termed "higher-level" processes, i.e., concepts, goals, etc., from "lower-level" processes, i.e., the ones termed "perceptual", and then one can easily object that those very conditions imply that "normal" cognition is necessarily unitary, in that sense. Merleau-Ponty argues along those lines, for example. And language makes it even more complex. Just how "abstract" is language? The controversy here is very nasty, between the Chomskians and the cognitive linguists... and that's simplifying the oppositions. I just can't give you a simple answer here, because really there is none. I guess you could say that unconscious operations, i.e., the processes that go on before cortical level VI or so, visually, are "perceptual", sort of, and that those past that are so imbued with "concepts" that one cannot differentiate further. But even that is too simple.

2) No. Go look up, for example, the whole enormous controversy which Jackson's paper about Mary started:

Jackson, F. "What Mary Didn't Know." The Journal of Philosophy 83, no. 5 (1986): 291-95

That's the point of view you're attempting to hold... believe me, there's lots of disagreement:

Bringsjord, S. "In Defence of Impenetrable Zombies." Journal of Consciousness Studies 2, no. 4 (1995): 348-51.

McGeer, V. "The Trouble with Mary." Pacific Philosophical Quarterly 84 (2003): 384-93.

Pelczar, M. "Enlightening the Fully Informed." Philosophical Studies 126 (2005): 29-56.

3) Um... no, this is just too much for me to deal with. Go read, e.g., the Churchlands (e.g., Churchland, P.S. "Epistemology in the Age of Neuroscience." The Journal of Philosophy 84, no. 10 (1987): 544-53), or something like Goldman, Block, Bringsjord, or some others, below. Let's see... functionalism has to do with abstracting processes from physical systems so that, for example, computers can be made equivalent, as processors, with neural dynamics, because, in the abstract, the information processing events, if you want to put it that way, are ultimately identical. So they argue. I actually don't agree with that particular claim, viz., equating digital computers and the central nervous system (CNS), on whatever level of abstraction... but that doesn't mean I have to be a dualist, just that I don't think that the fundamental processes, abstracted or not, in the CNS are the same as those in a digital computer. Which is the position I hold. Really, this is just too complex to go into here. Here's a few, very few, refs:

Anderson, J. A. "Human Symbol Manipulation within an Integrated Cognitive Architecture." Cognitive Science 29 (2005): 313-41.

Apolloni, B., D. Malchiodi, C. Orovas, and G. Palmas. "From Synapses to Rules." Cognitive Systems Research 3 (2002): 167-201.

Dik, S.C. "On the Notion 'Functional Explanation'." Belgian Journal of Linguistics 1 (1986): 11-52.

Gallese, V. "Embodied Simulation: From Neurons to Phenomenal Experience." Phenomenology and the Cognitive Sciences 4 (2005): 23-48.

Gruber, O., and T. Goschke. "Executive Control Emerging from Dynamic Interactions between Brain Systems Mediating Language, Working Memory and Attentional Processes." Acta Psychologica 115 (2004): 105-21.

Kahn, I., L. Davachi, and A. D. Wagner. "Functional-Neuroanatomic Correlates of Recollection: Implications for Models of Recognition Memory." The Journal of Neuroscience 24, no. 17 (2004): 4172-80.

Muller, N.G., O.A. Bartelt, T.H. Donner, A. Villringer, and S.A. Brandt. "A Physiological Correlate of the "Zoom Lens" of Visual Attention." The Journal of Neuroscience 23, no. 9 (2003): 3451-565.

Overgaard, M. Problems in the "Functional" Investigations of Consciousness [html]. University of Aarhus, 2000

Shallice, T. "Information-Processing Models of Consciousness: Possibilities and Problems." In Consciousness in Contemporary Science, edited by A.J. Marcel and E. Bisiach. New York, NY: Clarendon Press, 1988.

Sun, R. "Computation, Reduction, and Teleology of Consciousness." Journal of Cognitive Systems Research 1 (2001): 241-49.

Zelazo, P.R., and P.D. Zelazo. "The Emergence of Consciousness." In Consciousness: At the Frontiers of Neuroscience, edited by H.H. Jasper, L. Descarries, V.F. Castellucci and S. Rossignol, 149-65. New York, NY: Lippincott-Raven Press, 1998.

4) Well, I guess it means you're an interactive dualist; non-interactionism is beyond absurd, don't you think (it all runs on coincidence? um...)? Anyway, yes, well, your position is a very very old one, and I'm afraid that it's pretty much incomprehensible to me. I have problems believing in ghosts. I'm not going to give you references here, because this controversy is literally thousands of years old. If you want to look at some of the hundreds (thousands, more likely) of papers on materialism vs. dualism written in the last century or so, just go to your local academic library and dig in. Really, however, what I'd advise, before you immerse yourself in dualism and the philosophical controversy, is that you take a good look at the literature in cognition and neurology that's been accumulating since the early part of the last century (see below for a start). That might give you a bit of a different perspective, one which actually has some of that strange stuff, "data", to back it up, rather than only logic, which is always dependent on assumptions, which, if they're not based on data, are just, well, armchair speculations. Then we get into another controversy, i.e., what "data" is... and for that I highly recommend you look at this book: Kitcher, P. The Advancement of Science; Science without Legend, Objectivity without Illusions. New York, NY: Oxford University Press, 1993. Some more:

Reisberg, D. Cognition: Exploring the Science of the Mind. 1st ed. New York, NY: W. W. Norton & Company, Inc., 1997.

Johnson, M., and G. Lakoff. "Why Cognitive Linguistics Requires Embodied Realism." Cognitive Linguistics 13, no. 3 (2002): 245-63.

Goldman, A. I. (Ed.). Readings in Philosophy and Cognitive Science. Cambridge, MA: The MIT Press, 1995.

Dehaene, S., ed. The Cognitive Neuroscience of Consciousness. Edited by J. Mehler. 1-2 vols. Vol. 79, Cognition Special Issues. Cambridge, MA: The MIT Press, 2001.

Clark, A. Being There: Putting Brain, Body, and World Together Again. Cambridge, MA: The MIT Press, 1998.

Bringsjord, S. "Are We Evolved Computers? A Critical Review of Steven Pinker's How the Mind Works." Philosophical Psychology 14, no. 2 (2001): 227-43.

Block, N. "The Computer Model of the Mind." In Thinking, edited by L.R. Gleitman, D.N. Osherson and E.E. Smith. Cambridge, MA: The MIT Press, 1990.

Baars, B. J. A Cognitive Theory of Consciousness. 3rd ed. Cambridge, MA: Cambridge University Press, 1993.

But really this is barely the tip of the iceberg. Look at some of the references in the above, for more sources.

As for the "agony"... um, well, there's no avoiding the existentialist crises in this area, I'm afraid... once you get going here, you just have to make peace with the thin ice you'll walk on from here on out. Hey, it can be fun, too.

Steven Ravett Brown

back

(53) Judy asked:

Are roses red in the dark?

---

Yes unless they are being soaked in bleach, in which case they will be turning white.

Something can be described as being red if it looks red when viewed in daylight. It doesn't lose its redness just because we can't see it.

Shaun Williamson

back

(54) Katie asked:

The Rain in spain falls mainly on the plain. What would be a sufficient philosophical answer to this point? And can we give a sound answer?

---

This is not a philosophical question. Its a question about the weather and the geography of Spain. You would have to look at records of rainfall in different areas of Spain and classify these areas as either being plains or not plains.

Shaun Williamson

back

(55) Lauren asked:

I need help with all but any help would be appreciated:

1) In the dialogue Euthyphro, the title character and Socrates discuss different possible ways to define right and wrong. There are two definitions in particular that serve as the basis for the dialogue. What are these definitions? How do they differ? What fault does Socrates find with them and what conclusion should we draw from his objections? Finally, do you think religious authority should define right and wrong? Give an argument explaining why or why not.

2)The first book of Nicomachean Ethics attempts to explain how human beings are different from other things. Why does Aristotle need to draw this distinction and what does it have to do with his account of what is good? Explain how he defines "human being" and how this distinguishes humans from other living things. Aristotle goes on to say that certain people are unlikely to lead good lives. How does this follow from his definition? Do you think Aristotle is right? If not, would it be better to change the definition of "human being"? Give an argument defending your opinion.

3) In chapter 5 of Book X of the Nicomachean Ethics, Aristotle writes that "each pleasure is proper to the activity that it completes." He later goes on to call study and contemplation the greatest pleasures that humans can feel. To which activity are these pleasures proper, and how does this tie in to Aristotle's definition of "human being"? (Hint: to say that a pleasure is proper to some activity is to say that such a pleasure can only be felt by engaging in that activity.) Do you agree that study and contemplation are the greatest pleasures humans should feel? If not, which pleasures could be more proper to this distinctly human activity? Give an argument explaining your opinion.

4) A hedonist is someone who claims that the point of life is to experience pleasure and happiness and that these things define what is good. Is Aristotle a hedonist? What does he say is the ultimate goal of human life, and what does this goal have to do with what is good? If Aristotle's view is different from that of the hedonist, explain how. Who do you think has a better account of what is good? Give an argument defending your answer.

5) What is the difference between rational and non-rational virtues? Aristotle distinguishes between two different kinds of rational virtue. What are they? What is the Doctrine of the Mean, and to which of the two different kinds of rational virtue does it apply? Explain how one kind of rational virtue influences the other. Which kind of virtue does Aristotle say is most important in leading a good life? Do you agree with him? Give an argument explaining why or why not.

---

1) Socrates in Plato's Dialogue 'Euthyphro' asks Euthyphro what is pious (osion), and the first definition by him is 'pious is what is loved by the gods'. As Socrates distinguishes the difference between 'acting' (poiein) and being acted upon (paschein), he argues that because something is pious, therefore is loved by the gods and not because is loved by the gods is pious, namely piety is a good thing in itself, therefore the gods love it (9-10). After this comes the need of the second definition, namely 'what is piety'. And Euthyphro argues that it is a part of Justice (dikaiosyne) (11-12). Justice in dealings with gods, namely service to the gods, in other words honour and gratification. But this is also pleasing to the gods (14-15), or else loved by the gods. Therefore the definition of 'what is piety' is not answered. Through his dialectic Plato leads to the awareness and deeper understanding of the Forms, and as a result the moral values. In this way he challenges the moral standards of his time, which Euthyphro invokes, and proves that no moral value has to be taken for granted, even if is certified by religious and political authority.

2) As Aristotle distinguishes different parts in the soul of the beings, first he defines the rational part — which possesses reason (logon echon) and only the human beings possess — and the irrational part (alogon). Then he divides further the irrational part in the nutritive related to the nutrition of the body, the sensitive related to the sensation of the body, and the appetitive related to the appetites. The animals and the plants have both the nutritive and the sensitive, therefore he calls it vegetative, but the animals and the human being apart from that, have the appetitive part (Apart from Nicomachean Ethics I, 1102b you can see also On The Soul III 433b). The appetitive can share in reason for some people (continent) or can go against the reason (incontinent). How is related to reason this part (appetitive) through the imagination, Aristotle explains more (On The Soul 433a, b), that sometimes the rational part fights against the appetitive one through the imagination, and does not originate the movement of the body towards a certain sensational pleasure (e.g. continence), or the contrary (incontinence).

3) Since Aristotle defines human being a rational being, then the activity in accordance with rational nature of man and as a result proper to him, is the life according to intellect (kata noun vios), namely knowledge (episteme) and contemplation (theorein) (Nicomachean Ethics X 1178a). However, he defines human being also as a social being (politikon on), which means the activity that is in accordance with this aspect of man's nature is life in accordance with moral virtue of the Mean. The activities then that are proper to this twofold nature of the human being (rational and social being), are on the one hand knowledge and contemplation for the rational being, and on the other life in accordance with moral virtue of the 'Mean' for the social one, along with the pleasures which accompany all the a/m activities.

4) The virtue of the Mean means activity for the sake of the 'good' and the noble after choice and deliberation, namely noble deeds for noble ends, beyond pleasure and pain. Through reasoning and thought (prudence) one chooses the right action, being aware of the persons involved in the action, the means, the end and the consequences of the action. These noble deeds are not for the sake of the pleasure (hedone), but for the sake of the 'good', and besides are not always pleasant. Therefore we could not say that Aristotle is an hedonist.

5) The rational part of the soul is also twofold — just like the irrational part (vegetative, appetitive part) — namely the intellective (noetikon) and the deliberative part (bouleutikon), which corresponds to the volition or else will (boulesis). Since the virtue is associated with activity of the soul, then to each part of the rational part must correspond certain virtues (excellences). Therefore, there are two kinds of virtues, intellectual (dianoetikai) corresponding to the 'intellective' part, and moral or ethical (ethikai) corresponding to the 'deliberating' one, and as a matter of course to a man's character. For example, wisdom, understanding and prudence are intellectual virtues, while calmness and temperance are moral or ethical ones (Nicomachean Ethics I, 1102b). The doctrine of the Mean is applied more to the moral virtues, apart from prudence. Through prudence one chooses what is good and noble for the human being and the means of achieving it (Nicomachean Ethics VI, 1140 b), and through deliberation (volition) and choice one can become temperate or brave etc. after having exercised this activity of the soul. As for philosophic wisdom (sofia) and comprehension or pure intellect (nous), which is of divine origin, through contemplation one can grasp the first principles, and can lead a happy life.

Since life is activity and happy life consists both activities, actions in accordance with virtue (good man), and activity in accordance with intellect (nous) through contemplation on the principles of all things (wise). In this way one fulfils the composite nature of man, namely social and thinking being, and as result contribute to his happiness, since the being lives in accordance with its nature. You can see more in Aristotle's Nicomachean Ethics (1177a-1178b).

Nikolaos Bakalis

back

(56) Joshua asked:

Do cats like cat food because it tastes good to them the way steak tastes to us, or does it taste to them the way cat food tastes to us, only they like the taste?

---

Yes, well, this is a very good question which unfortunately no one knows how to answer. Very recently, it's been found that even between human beings, there are enormous variations in things like the number of cells in the eyes (in the retina) that respond to various colors, and that we either a) compensate for this variation in some way so that we all see the red of a fire engine, for example, the same, OR b) we don't see them as the same, but behave as if we do, because we simply learn to deal with whatever particular color it is the same way everyone else deals with the color they see on fire engines. Get it? So now you're asking about cats, well... there's just no way we know. If you're really interested in this go read:

Nagel, T. "What Is It Like to Be a Bat? " The Philosophical Review 83, no. 4 (1974): 435-50.

Watkins, M. "Seeing Red: The Metaphysics of Colours without the Physics." Australasian Journal of Philosophy 83, no. 1 (2005): 33-52.

And look up the "hard problem" on the web... but be prepared for some difficult reading.

Steven Ravett Brown

back

(57) Askphilosophers asked:

How can we be sure that we perceive color the same way? In other words, how do I know that the red I see looks the same as the red that you see?

We are taught from birth to identify red objects as red, but what if what someone calls red really looks green for example, yet they only call it red because that is what has been taught?

---

We can be pretty sure that some people experience colour differently, namely the colour blind. What it would be like to really worry about the colour experiences of the non-colour blind I am less sure. In the case of the colour blind something hangs on the difference in experience, i.e. they might inadvertently go to a funeral in a blue suit, or go through a traffic light mistaking red for amber. But does anything hang on whether the non-colour blind experience colour differently? Not really. So does it matter? I mean, is it a genuine living question, or just the sort of thing we do to divert ourselves fruitlessly in empty hours, like building a tower of wooden bricks and seeing how many we can remove before the thing falls down? Evidently this is an unanswerable question, but on what grounds would we regard it as both unanswerable and interesting?

David Robjant

back

(58) Dan asked:

I am a third year undergrad philosophy student. much as I enjoy the subject I can't help but feel that sometimes it is too theoretical. I am also a zen buddhist. zen is a philosophy of action. one of the key elements of zen is that the only way one can know reality is through directly experiencing reality by sitting in zazen (zen meditation — google it if you want). zen emphasises that words alone will never make us any happier or wiser and it seems to me that western analytic philosophy revels in words at the cost of the philosopher losing site about what she should actually do to make herself happier and wiser (i am assuming that those are two near universal and closely linked goals — happiness and wisdom). what defense can be made of the western analytic philosophical tradition's preoccupation with words at the cost of losing sight in what we should actually DO? of course we need words to learn what we should do but words alone will get us nowhere. we also need to do it!

---

Um, you don't want much, do you? You might take another look at Western philosophy, however. What does Socrates (via Plato) say about words... what underlay his debate with the Sophists? What does Aristotle say about the good life in the Ethics? Both of them emphasize the integration of action, thought and logos, right? Both are about attaining wisdom, yes? Perhaps you need to go back to your roots, rather than plunging into a tradition which is based on a culture different enough from ours that one needs years of immersion in it to fully understand it. Furthermore, the style of Zen which you're talking about (i.e., the Japanese adaptation of the Chinese version of Indian Buddhism) doesn't particularly emphasize action, but as you yourself state, sitting meditation. For an interesting Western take on the cultural roots of this practice, I recommend:

Koestler, A. The Lotus and the Robot. London, England: Hutchinson, 1966.

And and of course you have most likely read:

Watts, A.W. This Is It: And Other Essays on Zen and Spiritual Experience. New York, NY: Collier Books, 1971.

------. The Way of Zen. New York, NY: Mentor Books, 1964.

Ross, N.W., ed. The World of Zen: An East-West Anthology. New York: Random House, 1960.

Herrigel, E. Zen in the Art of Archery. Translated by R. F. C. Hull. New York, NY: Vintage Books, 1971.

Okakura, K. The Book of Tea. New York, NY: Dover Publications, Inc., 1964.

Interestingly enough, Watts died of a heart attack... perhaps a comment on the effectiveness of Zen re serenity, for Westerners. I particularly recommend Ross' book for a more general treatment of Zen in Japanese culture. And no, I don't think much of the various ideas of the martial "arts" as meditation... any action, or none, can be viewed in that manner. To put this more generally, after many years of exposure to those ideas, I have very mixed feelings about them. They serve particular roles in the cultures in which they're practiced... roles which do not fit our culture too well, by and large. We have our own, rather well thought out traditions (and while 4000 years — since Ptolemy and others borrowed from Egypt — is not 5000, still it's not insignificant either), and I think that before placing the Eastern and Western in conflict, which you are evidently pleased to do, and which is indeed commonly done, it would be much better to take from each what is appropriate to our particular and general situations and attempt to integrate them. You might think about the benefits of linguistic precision, and how that might integrate with action, for example. The sciences spring to mind as a particular example of the latter... and don't knock those unless you're prepared to abandon all plastic, metal (except the crudest... bye bye bicycle), leather (same), packaged and/or preserved food, transportation (aside from horses), medicine, books (yes, indeed)... shall I go on?

Steven Ravett Brown

back

(59) Jamie asked:

Why is symmetry inherently beautiful? (I do not require a base answer pertaining to the effect that symmetry has on us, I wish to know why this affect happens. What fundamental imago does symmetry satisfy? I realise this question is probably unanswerable.)

---

Two answers I could mention. One, that symmetry in human beings is a sign of health and is therefore (for evolutionary reasons) attractive in a mate. (This argument has limitations in that a) lots of attractive faces are less than symmetrical, b) we find symmetry attractive in lots of things other than human beings, where nothing is evolutionarily relevant). Two, which is the variety of Platonist answer better fitted to the excesses of Islamic Architecture, is that the perfection and order of symmetry reminds us of the Forms, hyper-real intellectual objects, the contemplation of which is argued to be essentially more satisfying than the pursuit of the 'fleeting' and 'unreal' objects of the senses. Plato would also think that the way in which we find symmetrical objects satisfying as to do with such beauty being an image (not sign) of harmony (or due proportion) between the three parts of the soul, which is how he pictured The Good for human beings. Thus for Plato beauty is a kind of picture of the good.

I might agree with Plato about the nature of some of the satisfactions we find in symmetry, but without thinking that his account captures the whole story. And I might add that there is room for some doubt about whether in fact symmetry is inherently beautiful, if it can in fact result in great ugliness (eg in brutalist architecture). It seems that it matters what is mirrored. Mirroring geometric forms can work, but proportion matters. So does colour, texture, animation, etc. Certainly, if one was trying to find a theory of beauty which explained all our practice with the word 'beauty', then 'because it is symmetrical' wouldn't work. And I would reject the thought that an arrangement of corpses would be any more beautiful for being symmetrical. So if by 'inherently beautiful' you mean 'beautiful whatever else applies' I am not convinced. But certainly the beauty of symmetry is one kind of beauty (beauty can attach to or 'inhere in' the symmetry), and symmetry is one strong component in the beauty of many things we find beautiful. But let us allow for the thought that the greatest beauty might be an arrangement of things poured out at random.

David Robjant

back

(60) Elise asked:

Why is the question of determinism important in a discussion of moral philosophy?

---

To be morally praised or blamed for doing something requires agency. Agency means that the agent has the freedom to reflect, deliberate and choose to do/ not do an act; namely the 'right' thing to do. Determinism holds that actions are effects determined by prior causes which are themselves effects of prior causes and so on. Such causes are viewed as being variously the effects of Gods decree, of the outcome of my essence, as the outcome of my nature or as determined by my genes and the like. The general charge is that determinism removes the freedom of agency. If no freedom of agency than I do not freely choose my actions. I cannot therefore be morally praised or blamed. If morality is about reflecting how I ought to act and having the freedom to do otherwise; determinism removes this as it maintains 'I' have no choice in the matter. My actions were determined. So it is maintained by thinkers in moral philosophy that determinism determines the collapse of Morality, praise and blame.

Martin Jenkins

back

(61) Jorge asked:

I am currently studying Berkeley and his idea that nothing exists. I was wondering if you could clear something up for me. I figured out from his argument that the material world only 'exists' in our minds but my concern is that if it exists in our minds shouldn't our minds exist thus negating the argument that nothing exists? Now if our minds are simply non-material things, then is Berkeley forced to subscribe to the idea that non-material things exist but material things do not?

---

You premise your reasoning on the view attributed to Berkeley that nothing exists thus creating the conditions for the contradiction you have identified. This premise is not true so the contradiction does not follow. Berkeley never wrote that nothing exists. He writes that a world full of furniture such as tables, chairs and mountains does exist. They are real but what exists is not material but Immaterial. Things exist insofar as they are sensations, ideas perceived by mind/ spirit. There is nothing more than the sensations: no substance or material primary qualities lying behind them. Obviously, ideas require a mind for their existence; they cannot exist without a mind. All that ontologically is, is mind /spirit. When not perceived ideas cannot be affirmed to exist by the individual mind/spirit. They do however; continue to be perceived by other minds/ spirits and, they are continuously perceived by the infinite mind/spirit that is God. I hope this goes some way to clearing your problem.

Martin Jenkins

back

(62) Dan asked:

I am a third year undergrad philosophy student. much as I enjoy the subject I can't help but feel that sometimes it is too theoretical. I am also a zen buddhist. zen is a philosophy of action. one of the key elements of zen is that the only way one can know reality is through directly experiencing reality by sitting in zazen (zen meditation — google it if you want). zen emphasises that words alone will never make us any happier or wiser and it seems to me that western analytic philosophy revels in words at the cost of the philosopher losing site about what she should actually do to make herself happier and wiser (i am assuming that those are two near universal and closely linked goals — happiness and wisdom). what defense can be made of the western analytic philosophical tradition's preoccupation with words at the cost of losing sight in what we should actually DO? of course we need words to learn what we should do but words alone will get us nowhere. we also need to do it!

---

You are completely right in one thing, namely in the thought that it is possible for verbal virtuosity to be a special kind of sterile ego-game, and that this is an ugly vice in what we call 'philosophy', just as it is everywhere.

But on the other hand I am a little doubtful about the way you think you've summed up an informative difference between 'western' and 'zen' philosophy as a contrast between an emphasis on theory (western) as against an emphasis on action (zen).

After all, in the Buddhist tradition, the key thing that needs to be adjusted, in order for the deed of enlightenment (or happiness and wisdom) to be attained, is wrong attitudes towards or habitual false theories about our experiences. There is the saying: before enlightenment chop wood, after enlightenment chop wood. It is important that much of what a Buddhist is supposed to 'do' differently concerns what one might call mental 'actions', eg attachment. It would be easy to imagine that you were accusing western philosophy of being 'theoretical' in the sense that it is active 'in the head', but on close examination much of what is important and distinctive about buddhist practice (as compared to competing religions) is just this (philosophical) concern with 'actions' in the head. In which respect philosophers of all stripes and Buddhists are very much alike, and Socrates and the Buddha much of a muchness: the examined life is the examined interior life. In addition, the Buddhist literature tradition (not excluding even Zen) is just choc-a-bloc with theoretical (in a perfectly good and useful way) epistemology and metaphysics — what is there and what is knowledge? — thought of a kind deeply recognisable to those familiar with greek philosophy. This contrast between 'active' zen and 'theoretical' western philosophy doesn't look very plausible.

I think the sort of point that's more fundamental to what you want to say here is that the contrast lies in this, that however active or intelligently concerned with action a philosopher is, his thinking may well be fruitless in a certain sense, fruitless in a sense that does not effect his insightfulness or his status as a good philosopher, and fruitless in a sense that Zen practice (ideally at least) isn't. Philosophical insight doesn't automatically make you happy or wise, whereas the right kind of practice ought to (in theory? ). (And there is a distinction here between the philosopher and the philosopher's life in a way that there isn't between a monk and his life). There may be something in this distinction with the fruits of the different practices. But I would prefer not to have it come simply to 'the difference is that Zen has the fruit of enlightenment', because this of itself will not help to explain to those uncomprehending of (or perhaps not interested in) such states what is distinctive about the way you went there, and it seems to me that this was what you wanted to achieve with your contrasting Zen and Philosophy.

David Robjant

back

(63) Joshua asked:

Do cats like cat food because it tastes good to them the way steak tastes to us, or does it taste to them the way cat food tastes to us, only they like the taste?

---

Well this is a scientific question not a philosophical one. Here is a simple experiment you can do. Offer your cat some steak and some of its favourite cat food. You will then see which one it eats first. Try this test on as many cats as possible before reaching a conclusion.

Shaun Williamson

back

(64) Meagan asked:

In one of my philosophy classes, I vaguely remember a theory about believing in a god and the risk you take if you don't. Wondering what the theory is called and what exactly it is. Thanks

---

I think its called Pascal's Wager. Blaise Pascal was a 17th century French philosopher and mathematician. He is a founder of probability theory and invented the mechanical calculator.

Do a search on Yahoo and you will probably be able to find the exact quote.

Shaun Williamson

back

(65) Adam asked:

Can you prove something exists without being able to see, hear, smell, or touch it?

---

Well, does the idea of seeing something exist? What about the idea of ideas? Perhaps you need to think about what "exist" might mean.

Steven Ravett Brown

back

(66) Justin asked:

I remember reading somewhere about William James's (or possibly Pierce's) categories of symbols. In one place a weather vane was mentioned as a type of symbol that actually is the thing it symbolizes. Can anybody point me to the primary source of this?

---

You probably want to read Peirce. He's not easy. I'd read some of his more basic stuff before I'd start on his thoughts about symbolisms. Here is a variety of sources:

Ketner, K. L. Peirce and Contemporary Thought: Philosophical Inquiries. Edited by V. M. Colapietro and V. G. Potter. 1st ed. Vol. 1, American Philosophy Series. New York, NY: Fordham University Press, 1995.

Peirce, C.S. "Some Consequences of Four Incapacities." In The Essential Peirce: Selected Philosophical Writings, edited by N. Houser and C. Kloesel, 28-55. Bloomington, IN: Indiana University Press, 1992.

------. "Questions Concerning Certain Faculties Claimed for Man." In The Essential Peirce: Selected Philosophical Writings, edited by N. Houser and C. Kloesel, 11-27. Bloomington, IN: Indiana University Press, 1992.

------. "Deduction, Induction, and Hypothesis." In The Essential Peirce: Selected Philosophical Writings, edited by N. Houser and C. Kloesel, 186-99. Bloomington, IN: Indiana University Press, 1992.

------. "On the Logic of Drawing History from Ancient Documents, Especially from Testimonies." In The Essential Peirce: Selected Philosophical Writings, edited by N. Houser, A. De Tienne, J. R. Eller, C. L. Clark, A. C. Lewis and D. B. Davis, 75-114. Bloomington, IN: Indiana University Press, 1998.

------. "What Makes a Reasoning Sound? " In The Essential Peirce: Selected Philosophical Writings, edited by N. Houser, A. De Tienne, J. R. Eller, C. L. Clark, A. C. Lewis and D. B. Davis, 242-57. Bloomington, IN: Indiana University Press, 1998.

------. "The Basis of Pragmaticism in the Normative Sciences." In The Essential Peirce: Selected Philosophical Writings, edited by N. Houser, A. De Tienne, J. R. Eller, C. L. Clark, A. C. Lewis and D. B. Davis, 360-71. Bloomington, IN: Indiana University Press, 1998.

Shanahan, M. "Perception as Abduction: Turning Sensor Data into Meaningful Representation." Cognitive Science 29 (2005): 103-34.

Then you could look at:

Houser, N., and C. Kloesel. The Essential Peirce: Selected Philosophical Writings. Vol. 1, The Peirce Edition Project. Bloomington, IN: Indiana University Press, 1991.

Houser, N., A. De Tienne, J. R. Eller, C. L. Clark, A. C. Lewis, and D. B. Davis. The Essential Peirce: Selected Philosophical Writings. Vol. II, The Peirce Edition Project. Bloomington, IN: Indiana University Press, 1998.

Peirce was quite brilliant and not always sound... but more so than most people think, I believe.

Steven Ravett Brown

back

(67) Tom asked:

I have a theory that there is no such thing as time travel, and there will never be a time-machine or means of travelling through times, purely because if we do invent one one day, surely we would have heard about them by now? Surely someone would've travelled back in time from the future to our time and presented themselves.

---

Look, why don't you go read some good sci-fi on time-travel. You'll find well worked-out theories, hypotheses, all illustrated with examples in the form of stories, some more and some less well-based in physics. You'll have fun, and you'll learn lots of ways of thinking about time-travel. Then, after you've spent 6 months or so doing that, you might go look at some serious philosophy on the subject of time, of which there is quite a bit, mostly very very technical and difficult.

Steven Ravett Brown

back

(68) Judith asked:

For me it is obvious that no one, a human being or other living organisms have souls. But why is it that I haven't heard about anyone else who chares this thought? and is there any famous philosophers that think this way?

---

The answer is obvious: because you haven't done any reading on this subject. Go here to get started:

http://www.infidels.org/news/atheism/overview.html

and,

http://www.simonyi.ox.ac.uk/dawkins/WorldOfDawkins-archive/index.shtml

I mean, just put "atheism" into Google, ok?

Steven Ravett Brown

back

(69) Daniel asked:

One for the aestheticists here: why do we enjoy listening to music that makes us feel quasi sadness (it's not real sadness after all because it's not directed at anything)? As a rule, do we not tend to avoid what makes us feel any kind of sadness?

---

Do we? Then why do we read books that make us feel sad? Poems? Why do we take pleasure in viewing paintings depicting sadness? As for it not being "real"... what a baldly confident assertion to make! I do think you might reflect on the absolute negation implied by the term "anything", above. Perhaps you might consider a short introduction to this rather well-developed field:

Anderson, A.K., and E. A. Phelps. "Expression without Recognition: Contributions of the Human Amygdala to Emotional Communication." Psychological Science 11, no. 2 (2000): 106-11.

Bachorowski, J., and M.J. Owren. "Production and Perception of Affect-Related Vocal Acoustics." Annals of the New York Academy of Science 1000 (2003): 244-65.

Balkwill, L-L., and W.F. Thompson. "A Cross-Cultural Investigation of the Perception of Emotion in Music: Psychophysical and Cultural Cues." Music Perception 17, no. 1 (1999): 43-64.

Barwell, I. "How Does Art Express Emotion?" The Journal of Aesthetics and Art Criticism 45, no. 2 (1986): 175-81.

Blood, Anne J. , and Robert J. Zatorre. "Intensely Pleasurable Responses to Music Correlate with Activity in Brain Regions Implicated in Reward and Emotion." Proceedings of the National Academy of Sciences USA 98, no. 20 (2001): 11818-23.

Brower, C. "A Cognitive Theory of Musical Meaning." Journal of Music Theory 44, no. 2 (2000): 323-79.

Budd, M. "Music and the Communication of Emotion." The Journal of Aesthetics and Art Criticism 47, no. 2 (1989): 129-38.

------. "Musical Movement and Aesthetic Metaphors." British Journal of Aesthetics 43, no. 3 (2003): 209-33.

Collier, W.G., and T.L. Hubbard. "Musical Scales and Evaluations of Happiness and Awkwardness: Effects of Pitch, Direction, and Scale Mode." American Journal of Psychology 114, no. 3 (2001): 355-75.

Cowie, R., and R.R. Cornelius. "Describing the Emotional States That Are Expressed in Speech." Speech Communication 40 (2003): 5-32.

Dalla Bella, S., I. Peretz, L. Rousseau, and N. Gosselin. "A Developmental Study of the Affective Value of Tempo and Mode in Music." Cognition 80 (2001): B1-B10.

Davidson, R. J. "Darwin and the Neural Bases of Emotion and Affective Style." Annals of the New York Academy of Science 1000 (2003): 316-36.

Davies, S. "Kivy on Auditors' Emotions." The Journal of Aesthetics and Art Criticism 52, no. 2 (1994): 235-36.

Dempster, D. "How Does Debussy's Sea Crash? How Can Jimi's Rocket Red Glare?: Kivy's Account of Representation in Music." The Journal of Aesthetics and Art Criticism 52, no. 4 (1994): 415-28.

Dolan, R.J., and P. Vuilleumier. "Amygdala Automaticity in Emotional Processing." Annals of the New York Academy of Science 985 (2003): 384-55.

Ekman, P. "An Argument for Basic Emotions." Cognition and Emotion 6, no. 3/4 (1992): 169-200.

Gabrielsson, A., and P.N. Juslin. "Emotional Expression in Music Performance: Between the Performer's Intention and the Listener's Experience." Psychology of Music 24 (1996): 68-91.

Gabrielsson, A., and E. Lindstrom. "The Influence of Musical Structure on Emotional Expression." In Music and Emotion: Theory and Research, edited by P.N. Juslin and J.A. Sloboda, 223-48. Oxford, England: Oxford University Press, 2001.

Gabrielsson, A., and S.L. Wik. "Strong Experiences Related to Music: A Descriptive System." Musicae Scientiae 7, no. 2 (2003): 157-217.

Goldman, A. "Emotions in Music (a Postscript)." The Journal of Aesthetics and Art Criticism 53, no. 1 (1995): 59-69.

Hejmadi, A., R. J. Davidson, and P. Rozin. "Exploring Hindu Indian Emotion Expressions: Evidence for Accurate Recognition by Americans and Indians." Psychological Science 11, no. 3 (2000): 183-87.

Juslin, P.N., and J.A. Sloboda, eds. Music and Emotion: Theory and Research. Edited by R. J. Davidson, P. Ekman and K.R. Scherer, Series in Affective Science. Oxford, England: Oxford University Press, 2001.

Kappas, A., U. Hess, and K.R. Scherer. "Voice and Emotion." In Fundamentals of Nonverbal Behavior, edited by R.S. Feldman and B. Rime, 200-38. New York, NY: Cambridge University Press, 1991.

Kivy, P. "How Music Moves." In What Is Music?: An Introduction to the Philosophy of Music, edited by P. Alperson, 147-63. Philadelphia, PA: Pennsylvania State University Press, 1994.

------. Introduction to a Philosophy of Music. Oxford, England: Clarendon Press, 2002.

Koelsch, S., E. Kasper, D. Sammler, K. Schulze, T. Gunter, and A.D. Friederici. "Music, Language and Meaning: Brain Signatures of Semantic Processing." Nature Neuroscience (2004).

Krumhansl, C.L. "Music: A Link between Cognition and Emotion." Current Directions in Psychological Science 11, no. 2 (2002): 45-50.

Levinson, J. "Music and Negative Emotion." Pacific Philosophical Quarterly 63 (1982): 327-46.

------. "Musical Expressiveness." In The Pleasures of Aesthetics : Philosophical Essays, 90-125. Ithaca, NY: Cornell University Press, 1996.

McGaugh, J.L. "The Amygdala Modulates the Consolidation of Memories of Emotionally Arousing Experiences." Annual Review of Neuroscience 27 (2004): 1-28.

Meyer, L.B. Emotion and Meaning in Music. Chicago, IL: The University of Chicago Press, 1956.

Panksepp, J. "The Emotional Sources Of "Chills" Induced by Music." Music Perception 13, no. 2 (1995): 171-207.

Robinson, J. "The Expression and Arousal of Emotion in Music." The Journal of Aesthetics and Art Criticism 52, no. 1 (1994): 13-22.

------. "Musical Meaning and Expression." The Journal of Aesthetics and Art Criticism 54, no. 3 (1996): 307-09.

Scherer, K.R. "Vocal Affect Expression: A Review and a Model for Future Research." Psychological Bulletin 99, no. 2 (1986): 143-65.

------. "The Neuropsychology of Emotion." In Psychological Models of Emotion, edited by J. Borod, 137-62. New York, NY: Oxford University Press, 2000.

Scherer, K.R., and M.R. Zentner. "Emotional Effects of Music: Production Rules." In Music and Emotion: Theory and Research, edited by P.N. Juslin and J.A. Sloboda, 361-92. Oxford, Great Britain: Oxford University Press, 2001.

Scherer, K.R., M.R. Zentner, and A. Schacht. "Emotional States Generated by Music: An Exploratory Study of Music Experts." Musicae Scientiae Spec Issue (2002): 149-71.

Schwartz, D.A., C.Q. Howe, and D. Purves. "The Statistical Structure of Human Speech Sounds Predicts Musical Universals." The Journal of Neuroscience 23, no. 18 (2003): 7160 -68.

Scruton, R. "Understanding Music." Ratio XXV, no. 2 (1983): 97-120.

Sloboda, J.A. "Empirical Studies of Emotional Response to Music." In Cognitive Bases of Musical Communication, edited by M. Riess-Jones and S. Holleran. Washington, D.C.: APA, 1996.

------. "Musical Performance and Emotion: Issues and Developments." In Music, Mind, and Science, edited by S.W. Yi, 220-38. Seoul, Korea: Seoul National University Press, 1999.

Solomon, R.C. "Emotions, Feelings and Contexts: A Reply to Robert Kraut." Dialogue XXIX (1990): 277-84.

Sparshott, F. "Music and Feeling." Journal of Aesthetics and Art Criticism 52, no. 1 (1994): 23-35.

Tervaniemi, M., and E. Brattico. "From Sounds to Music: Towards Understanding the Neurocognition of Musical Sound Perception." Journal of Consciousness Studies 11, no. 3-4 (2004): 9-27.

Steven Ravett Brown

back

(70) Kristin asked

How shall I determine my purpose in life? Once determined, how do I understand it? Do we all know our "purpose"; should we all know our "purpose; what if we never know our "purpose"? Are we then failures as human beings? Or have we just "missed out" on something?

---

Kristin there is no way to know what you purpose in life is. There is nothing hidden, waiting for you to find it. It is more a question of you deciding what you want to do with your life and what sort of person you want to be. Its all a question of choice and that can be a frightening thing but also a liberating one. Some people have a very clear idea of what they want to do with their life, some people never have a clear idea about how they should lead their lives. That doesn't mean they are failures as people. It just means that life is more difficult for them. At various times in your life you may change your mind about what you want to do and what you want to become. A good place to start is to think about what sort of people you admire and then to think about how you could become like them.

Shaun Williamson

back

(71) Matthew asked:

Some people would say that philosophy cannot give definite answers to the questions that it asks, but that its value lies in the experience of attempting to answer these questions. As someone who takes philosophy seriously, I find this view attractive but I also have a gut feeling that something's not quite right about it. What do you think of this view?

---

If we cannot give definite answers to philosophical questions then it seems to me that philosophy is a waste of time. However I also believe that there are no philosophical questions. The real problem is how you can get to a position where you can see that there are no philosophical questions and therefore you don't need to find answers to them. Here is a quotation from my favourite non-philosopher, Wittgenstein. 'Philosophy is a battle against the bewitchment of our intelligence by means of our own language'.

Shaun Williamson

back

(72) Helen asked:

Do you think that Ask A Philosopher! might aid people in plagiarism? i.e. They ask rather than pursue for themselves the questions on a take home test, essay or project?

---

Indeed I do. That's why I try very hard not to provide answers to questions that are clearly invitations to plagiarize.

Steven Ravett Brown

back

(73) Shirley asked:

From a philosophical viewpoint--how does one decide who regulates science? Do we put the brakes on cloning a human being or do we let scientists do what they do--discover!

---

The nature of science requires discussion, challenge to theory, attempts to prove and to disprove hypotheses. There is, and can be, no control over this except by consensus. If you want control over cloning because of the publicity over it, let's try first to get control over something much more concrete, more dangerous, and more easily traceable: the manufacture and sale of small arms to anyone in the world who wants them. How about that first? Then we can move on to something like the control of biological weapons. After all that, maybe starvation? What do you think? I think cloning should be far down the list, myself.

Steven Ravett Brown

back

(74) Cristina asked:

Why does Alan Turing think machines think?

---

Well, first, Turing is dead, so he doesn't think anything. Second, he said very explicitly that machines probably never would be able to think like human beings. However, his intellectual descendants, some of them, do indeed believe, and believe is the operative word here since there is currently no testable hypothesis as to how consciousness might be generated by matter (which doesn't mean that there won't or can't be), that machines will be able to think like a human being.

Steven Ravett Brown

back

(75) Paul asked:

if we had eight fingers would our concept of mathematical thought be different? (e.g. replace decimal & metric systems with 8 based systems).

---

I think it's quite likely. 10 is not a particularly great base for arithmetic... 12 is much better, since it's divisible by four numbers instead of just two. There's certainly no reason to prefer one base over another, everything else being equal, except for factors like: how many separate numerals can one easily memorize (10, 12, what's the difference? ); how many divisors does the base have (so fractions and, um, "decimals" come out more easily). Base 2 might be awkward because of the length of the numbers required for simple calculations... and so forth.

Steven Ravett Brown

back

(76) Gwen asked:

To what extent can art be defined as knowledge in a non-conceptual form?

---

I don't consider art to be primarily about "knowledge", myself. Perhaps you might think about what, precisely, you mean by the terms "art" and "knowledge". Are you actually referring to non-verbal thinking when you say "art"? Let me put it this way, as a rough start: one might say that knowledge has to do with the accuracy with which we match belief and facts... that's pretty rough, but we can start there. Now, how does art, viz., painting, music, novels, relate to that in ways that non-artistic activities corresponding to those (e.g., photography not considered art, sounds meant merely to designate, words meant only as factual descriptions — in other words, uses of the same media in ways we do not consider art) do not?

You might look here, for a bit of the tip of this iceberg:

Dienes, Z., and J. Perner. "A Theory of Implicit and Explicit Knowledge." Behavioral and Brain Sciences 22, no. 735-808 (1999).

Gentner, D. "Structure-Mapping : A Theoretical Framework for Analogy." Cognitive Science 7 (1983): 155-70.

Kepes, G., ed. Education of Vision. 3 vols. Vol. 1, Vision + Value Series. New York, NY: George Braziller, 1965.

------, ed. Structure in Art and in Science. 3 vols. Vol. 2, Vision + Value Series. New York, NY: George Braziller, 1965.

Zacks, J.M. "Using Movement and Intentions to Understand Simple Events." Cognitive Science 28 (2004): 979-1008.

Arnheim, R. Art and Visual Perception: A Psychology of the Creative Eye. Berkeley, CA: University of California Press, 1974.

------. New Essays on the Psychology of Art. Berkeley, CA: University of California Press, 1986.

Barwell, I. "How Does Art Express Emotion? " The Journal of Aesthetics and Art Criticism 45, no. 2 (1986): 175-81.

Cowie, R., and R.R. Cornelius. "Describing the Emotional States That Are Expressed in Speech." Speech Communication 40 (2003): 5-32.

Davidson, R. J. "Darwin and the Neural Bases of Emotion and Affective Style." Annals of the New York Academy of Science 1000 (2003): 316-36.

Dempster, D. "How Does Debussy's Sea Crash? How Can Jimi's Rocket Red Glare? : Kivy's Account of Representation in Music." The Journal of Aesthetics and Art Criticism 52, no. 4 (1994): 415-28.

Fields, C. "The Role of Aesthetics in Problem Solving: Some Observations and a Manifesto." Journal of Experimental & Theoretical Artificial Intelligence 16, no. 1 (2004): 41-55.

Gasche, R. The Tain of the Mirror. Cambridge, MA: Harvard University Press, 1986.

Goodman, N. Languages of Art. 2nd ed. Indianapolis, IN: Hackett Publishing Company, 1976.

Levinson, J. Music, Art, and Metaphysics: Essays in Philosophical Aesthetics. Ithaca, NY: Cornell University Press, 1990.

London, J. "Musical and Linguistic Speech Acts." The Journal of Aesthetics and Art Criticism 54, no. 1 (1996): 49-64.

Sontag, S. Against Interpretation. New York, NY: Picador USA, 1990.

Steven Ravett Brown